Sie sind auf Seite 1von 27

Centro de Estudios Aula Neptuno - GRANADA

www.aulaneptuno.com

Ver.: 10/2001 7. Una mujer de 84 aos ingresa en urgencias procedente de una clnica en donde se la encontr por la noche en su cama con confusin e hipotensin. En la sala de urgencias se le mide una temperatura de 37,7C, una presin arterial de 70/20 mm.Hg. y un pulso de 160 lpm. No responde a estmulos, pero no presenta ningn otro signo fsico. Cul de las siguientes opciones indicara que la sepsis es la causa probable de la confusin y la hipotensin de esta paciente, contrastando con otras causas de hipotensin? 1. Una escasa resistencia vascular perifrica y un elevado gasto cardiaco. 2. Una gran resistencia perifrica y un reducido gasto cardiaco. 3. Una presin en cua de los capilares pulmonares de 26 mm.Hg. antes de administrarle lquidos. 4. Un pH en sangre arterial de 7,1. 5. Una concentracin de lactato srico de 22 mmol/L. 8. Paciente de 65 aos, esplenectomizado en la juventud tras traumatismo abdominal, consulta por haber comenzado, aproximadamente 12 horas antes, con fiebre de inicio brusco, dolor de caractersticas pleurticas en el hemitrax izqdo. y disnea progresiva. En el examen inicial impresiona de gravedad, objetivndose hipotensin e importante distrs respiratorio. Qu microorganismo es con mayor probabilidad responsable del cuadro clnico del paciente? 1. 2. 3. 4. 5. 9. Legionella. Klebsiella. Neumococo. St.aureus. Pseudomonas.

1.

Un recin nacido presenta dificultad respiratoria, distensin de las venas del cuello, hipotensin, hiperresonancia y disminucin del murmullo vesicular en un lado del trax, as como enfisema subcutneo. El diagnstico ms probable es: 1. 2. 3. 4. 5. Enfermedad de la membrana hialina. Neumona estafiloccica. Neumotrax y neumomediastino. Atelectasia primaria. Hernia diafragmtica.

2.

El calostro tiene todas las siguientes propiedades, excepto: 1. 2. 3. 4. Tiene ms grasas que la leche madura. Ms protenas que la leche madura. Ms rico en vitamina A que la leche madura. Mayor concentracin de sodio y potasio que la leche madura. 5. Tiene anticuerpos protectores. Cmo definiras al llamado rin de Ask Upmark? 1. 2. 3. 4. 5. Es una enfermedad qustica renal. Es una variante del rin en herradura. Es una malrotacin renal acentuada. Es una hipoplasia renal segmentaria. Es una variante de rin ectpico.

3.

4.

Cul de estos valores NO es signo de mal estado neonatal segn el APGAR? 1. 2. 3. 4. 5. Respiracin irregular. FCF > 120. Arcadas negativas. Flacidez muscular. Cianosis.

5.

A las 24 horas de vida, un recin nacido comienza a desarrollar un cuadro de dificultad respiratoria, letargia, hipotensin y fiebre. Ha presentado un episodio convulsivo. En el hemocultivo y en el tracto genital de la madre se asla un coco gram positivo que produce el factor CAMP. Diagnstico etiolgico: 1. 2. 3. 4. 5. Estreptococo ppyogenes. Estafilococo aureus. Estreptococo agalactiae. Estreptococo milleri. Estafilococo epidermidis.

La resistencia de los proteus a la accin de los antibiticos se basa en su capacidad para: 1. 2. 3. 4. 5. Adherencia por sus pilli. Desdoblar la urea. Moverse mediante flagelos. Producir betalactamasas. Reproducirse en pH cido.

10.

En cul de los siguientes restaurantes es ms probable que se produzca una infeccin por Bacillus cereus? 1. 2. 3. 4. 5. Casa Evaristo, Matanzas y Chacinas. Restaurante Soon Lin Tao. Restaurant A la Cuisine Bordelaise. Restaurante vegetariano Races y Puntas. Trattoria Porto Cino.

6.

Un paciente de 65 aos debuta de forma aguda con dolor en el hombro, malestar, fiebre, escalofros y aspecto txico. A la exploracin se comprueba eritema en la zona dolorosa y un intenssimo dolor a la palpacin de la zona. En pocas horas, la lesin progresa de forma notable en extensin, apareciendo edema pardo y mayor afectacin del estado general. Qu cuadro sospechas? 1. 2. 3. 4. 5. Osteomielitis aguda. Eritrodermia. Sndrome del shock txico. Imptigo no ampolloso. Fascitis necrotizante.

11.

La presencia de una artritis de rodilla en un nio de 14 meses, con lquido sinovial purulento, hace pensar como germen ms probable en: 1. 2. 3. 4. 5. Aureus. Rubeola. Agalactiae. Haemophilus. Gonococo.

-1-

Centro de Estudios Aula Neptuno - GRANADA


www.aulaneptuno.com

Ver.: 10/2001 da.Cul de los signos y sntomas siguientes es MENOS sugerente del diagnstico de psitacosis? 1. 2. 3. 4. 5. 17. Escalofros con temblores y fiebre de 40,6 C. Cefalea de gran intensidad. Tos no productiva. Rigidez de espalda y cuello. Diarrea.

12.

Paciente de 83 aos en estado grave de malnutricin es enviado al servicio de urgencias con un cuadro de tos y espectoracin ftida. En la Rx de trax se aprecia una zona redondeada de condensacin en base derecha y con nivel hidroareo en el interior. Cul de los siguientes enunciados consideras MENOS probable en la evolucin del paciente: 1. En un 20% de los casos se presentar bacteriemia con alta probabilidad de CID. 2. Es frecuente la evolucin a necrosis y empiema. 3. La mejora del paciente ocurrir empleando clindamicina i.v. 4. Es caso de I. renal acompaante no hay que ajustar la dosis de dicho antibitico. 5. La patogenia ms probable de este caso es por aspiracin de contenido orofarngeo.

No es cierto en el Sarcoma de Kaposi de los pacientes con SIDA: 1. Es la neoplasia ms frecuente. 2. Las lesiones cutneas semejan trayectos vasculares. 3. Es casi exclusivo de los homosexuales. 4. La afectacin pulmonar se distingue de la producida por Pn.carinii porque produce ndulos y derrame pleural. 5. Las lesiones cutneas se localizan en la mitad inferior del cuerpo.

13.

Una de las siguientes afecciones no es propia de la sfilis secundaria: 18. 1. 2. 3. 4. 5. Meningitis. Glomerulonefritis. Aortitis. Hepatitis. Uvetis.

Seala la falsa sobre las repercusiones orgnicas de la infeccin por VIH: 1. La biopsia ganglionar no est indicada en la linfadenopata que ocurre en fases precoces, salvo que se asocie a aumento rpido de tamao, consistencia anormal o coalescencia de ganglios linfticos. 2. La afectacin rectal ms frecuente son las lceras y erosiones secundarias a infeccin por el virus herpes simple. 3. La anemia secundaria a la infeccin por parvovirus B19 responde al tratamiento con inmunoglobulinas intravenosas. 4. La enfermedad cardiaca, aunque relativamente frecuente en las necropsias, no suele suponer un problema clnico frecuente en los pacientes con SIDA. 5. La causa ms frecuente de prdida rpida de la visin son los exudados algodonosos producidos por el propio VIH.

14.

Dentro de las localizaciones tpicas de la tuberculosis extrapulmonar es muy rara la aparicin de una de las siguientes: 1. 2. 3. 4. 5. Escrfula. Gastritis. Espondilitis. Coriorretinitis. Peritonitis.

15.

Un paciente de 28 aos consult por fiebre y dolor torcico. Es ADVP desde hace aos, y ha tenido hepatitis y mltiples episodios febriles. Llevaba una temporada sintindose mal, con fiebre y tos seca. Una semana antes del ingreso comenz con dolor retroesternal opresivo e inconstante, fiebre alta y disnea de esfuerzo. La exploracin fsica puso de manifiesto dermatitis seborreica. Se palparon adenopatas cervicales mltiples, alguna de ellas del tamao de un garbanzo. El resto de la exploracin y la analtica estaban dentro de la normalidad, salvo una discreta elevacin de las transaminasas. Los anticuerpos anti-VIH (anti-env) resultaron positivos, siendo negativos los antigag. La radiografa de trax mostraba adenopatas paratraqueales y mediastnicas. La reaccin con Mantoux con 5 U.de PPD dio una induracin de 10 mm. Cul es el dgto. ms probable en este paciente? 1. 2. 3. 4. 5. Infeccin por MAI. Infeccin por M.tuberculosis. Criptococosis. Nocardiosis. Toxoplasmosis.

19.

Un hombre de 29 aos que anteriormente estaba sano y que no toma ningn medicamento tiene desde hace cuatro semanas un poco de fatiga, sudoracin nocturna y elevaciones de la temperatura hasta 38C, pero ha podido seguir trabajando como vendedor. En la exploracin fsica se le aprecian linfadenopatas generalizadas con ganglios duros e indoloros. Cul de las siguientes afirmaciones es correcta? 1. Un ttulo de 1:64 en la prueba de SabinFeldman confirmara que la causa de la linfadenopata es la toxoplasmosis. 2. Un test ELISA positivo para el VIH en una ocasin confirmara que el VIH es el causante de la linfadenopata. 3. Una biopsia ganglionar y un hemocultivo en los que se encontrase Mycobacterium avium intracellullare indicara que muy probablemente este paciente tiene SIDA. 4. Una prueba Western-Blot positiva despus de varios positivos de la prueba de ELISA confirmara que este paciente tiene SIDA. 5. La determinacin de la proporcin CD4/CD8 en los leucocitos de sabgre perifrica reduci-

16.

Una mujer de 28 aos de edad, que trabaja en una compaa de procesamiento de aves,presenta una enfermedad febril agu-

-2-

Centro de Estudios Aula Neptuno - GRANADA


www.aulaneptuno.com

Ver.: 10/2001 24. Existe una meningoencefalitis amebiana primitiva casi siempre mortal, que se adquiere bandose en piscinas climatizadas y charcas de aguas templadas. Qu ameba la produce? 1. 2. 3. 4. 5. 25. Entamoeba histoltica. Entamoeba coli. Entamoeba hartmani. Endolimax nana. Naegleria fowleri.

ra las opciones de dgto.diferencial. 20. Paciente es traido a urgencias por deterioro progresivo del nivel de consciencia y episodio de convulsiones generalizadas. Segun su acompaante desde hace 24-48 h comenzo con alteracion del caracter, cefalea y malestar general. Se realizo tac craneal que mostro lesiones hipodensas en region basal de lob. Temporales. Se realizo puncion lumbar, con el siguiente resultado:lcr discretamente aumentado de presion, de aspecto normal. 800 Leucocitos con 10% pmn, 70% de linfocitos. 80 Mg/dl ( suero 90 mg/dl). Sin proteinorraquia. Gram negativo.Cual de los siguientes tratamientos estaria indicado: 1. 2. 3. 4. 5. 21. Aciclovir. Ceftriaxona + metronidazol. Ganciclovir. Dexametasona. Pirimetamina + sulfadiacina.

En referencia al diagnstico del ttanos, slo una de las siguientes es CORRECTA: 1. El estudio del LCR suele ser normal. 2. No est indicada la realizacin de cultivos de la herida. 3. C.tetani no aparece en heridas de pacientes sin ttanos aunque con frecuencia no se detecta tampoco en pacientes con ttanos. 4. El diagnstico es eminentemente clnico y no debe concederse importancia a la anamnesis sobre vacunaciones previas a la hora de emitir un juicio sobre un cuadro sospechoso de ttanos. 5. En el diagnstico diferencial con otros procesos que cursan con rigidez, la presencia de espasmos importantes a nivel carpopedal apoya definitivamente el diagnstico de ttanos.

El tratamiento de eleccin de la triquinosis es con: 1. 2. 3. 4. 5. Dietilcarbamacina. Metronidazol. Pentamidina. Tiabendazol. Paromomicina. 26.

22.

Cul de los siguientes ttos. deberas utilizar en primer lugar en un paciente diagnosticado de Colitis Pseudomembranosa? : 1. 2. 3. 4. 5. Colestiramina oral. Bismuto oral. Vancomicina + colestiramina orales. Metronidazol oral. Vancomicina oral.

Un ADVP con fiebre de dos semanas de evolucin, poliadenopatas, candidiasis oral y deterioro progresivo, es diagnosticado de infeccin por VIH y TBC por una tincin de ZiehlNielsen de un aspirado ganglionar. La radiografa de trax es normal y el PPD es negativo. Seala lo falso de la TBC en relacin con la infeccin por VIH: 1. La TBC se produce en ms del 50% de infectados previamente (PPD +) que tienen infeccin por VIH y no reciben profilaxis con isoniacida. 2. La TBC es una situacin clinica definitoria de SIDA en el contexto de la infeccin VIH. 3. El PPD da un elevado nmero de falsos negativos. 4. El tratamiento anti-tuberculoso debe prolongarse ms de doce meses en los pacientes VIH +. 5. La TBC de los VIH + suele ser por reactivacin de un foco tuberculoso antiguo.

23.

Un hombre de 62 aos fue trado al hospital por fiebre prolongada. No tena antecedentes de enfermedad y contaba que haba comenzado tres semanas antes con fiebre por las tardes. La temperatura suba hasta 39C y se acompaaba de cefalea, malestar general y dolores articulares. Las aglutinaciones a salmonella resultaron positivas, as que su sobrino mdico le trat con cloramfenicol sin xito. Se qued inapetente, dbil y observ que perda peso. La exploracin mostr palidez y prdida parcial de la masa muscular. No tena adenopatas y la auscultacin cardiopulmonar fue normal. Se palpaba esplenomegalia a 4 cm.del reborde costal. Tena 3.100 leucocitos/mm3; Hb de 9,2 g/dl; plaquetas 65.000/mm3; GOT 57 U/L; GPT 100 U/L; Albmina 2,6 g/dl e hipergammaglobulinemia. Todos los hemocultivos y una determinacin de anticuerpos anti VIH resultaron negativos. Una vez realizado el dgto. diferencial, qu exploracin te parece ms apropiada? 1. 2. 3. 4. 5. TAC abdominal. Ecografa abdominal. Linfografa. Puncin-aspiracin del bazo. Laparotoma y esplenectoma.

27.

Indica la respuesta falsa sobre las infecciones estafiloccicas: 1. En la endocarditis por estafilococo aureus son signos de mal pronstico la infeccin de la vlvula artica, la aparicin de insuficiencia cardiaca incontrolable o la participacin del sistema nervioso central. 2. El tratamiento de las infecciones cutneas ms superficiales no requiere el uso de antibiticos por va sistmica. 3. El sndrome estafiloccico de la piel escaldada puede presentarse, adems de en nios menores de 5 aos, en adultos inmunodeprimidos o con insuficiencia renal. 4. La determinacin del ttulo de anticuerpos anti cido teicoico es el nico dato en el que de-

-3-

Centro de Estudios Aula Neptuno - GRANADA


www.aulaneptuno.com

Ver.: 10/2001 3. Vrtebras dorsales. 4. Subtrocantreas de fmur. 5. Cuello de fmur. 34. La presencia de arco doloroso es muy sugestiva de: 1. 2. 3. 4. 5. 35. Tendinitis del supraespinoso. Artritis del codo. Artrosis de cadera. Tendinitis aqulea. Bursitis olecraniana.

bemos basarnos para diferenciar entre bacteriemia o sepsis y tomar las decisiones teraputicas oportunas. 5. La meticilina no suele usarse frecuentemente en el tratamiento de las infecciones estafiloccicas por la elevada incidencia de nefritis intersticial provocada por dicho antibitico. 28. Sobre las caractersticas microbiolgicas de los cocos indica la respuesta errnea: 1. Los enterococos crecen en medios con bilisesculina y tambin con cloruro sdico al 6,5%. 2. Los estafilococos son catalasa positivos y los estreptococos, catalasa negativos. 3. El estafilococo aureus se diferencia de los dems estafilococos por ser coagulasa positivos. 4. Dentro de los estreptococos beta hemolticos se incluyen el pyogenes, al agalactiae, el viridans y los enterococos. 5. La capacidad para utilizar maltosa diferencia al meningococo del gonococo. 29. En una embarazada de 30 aos de edad sin otros antecedentes de inters, y con PPD positivo, lo ms recomendable sera: 1. Realizar quimioprofilaxis con isoniacida. 2. Postponer la quimioprofilaxis con isoniacida hasta despus del parto. 3. Repetir el PPD 7 das despus. 4. Realizar tratamiento antituberculoso completo para impedir la transmisin al feto. 5. Ofertar interrupcin vountaria del embarazo. 30. La llamada neumona plasmocelular de los prematuros est relacionada con uno de los siguientes agentes: 1. 2. 3. 4. 5. 31. Virus del sarampin. Bordetella pertussis. Pneumocystis carinii. Aspergillus fumigatus. Estrongyloides stercolaris.

Un individuo que ha tenido que llevar muletas durante 6 meses por mltiples fracturas en fmur y tibia derechas comienza a notar cierta dificultad para la rotacin externa del hombro izquierdo, as como diestesia en la cara superoexterna del mismo. Cul de las siguientes crees que es la lesin responsable? : 1. 2. 3. 4. 5. N. torcico mayor. N. supraescapular. N. radial. N. circunflejo. Tendinitis del supraespinoso.

36.

Cul de las siguientes fracturas de ctilo debe recibir tratamiento conservador?: 1. Fractura de la ceja posterior con inestabilidad de la articulacion 2. Fractura transversal alta 3. Fractura con fragmento intraarticular 4. Fractura con desplazamiento menor de 2 mm. 5. Fractura de ambas columnas con desplazamiento de la posterior

37.

Un estudiante de 21 aos con una pseudoartrosis dolorosa de escafoides carpiano sin signos degenerativos debe ser tratado mediante: 1. 2. 3. 4. 5. Observacion Inmovilizacion mediante escayola Estiloidectomia radial Injerto oseo y fijacion interna Artrodesis radiocarpiana

Uno de los siguientes tendones musculares NO quedara afectado en caso de una Epicondilitis o "Codo de tenista" 1. 2. 3. 4. 5. Cubital posterior. Ancneo. Supinador corto. Flexor comn de los dedos. Radial.

38.

La osteomielitis hematogena aguda puede complicarse en el adolescente con una artritis, cuando el foco infeccioso se encuentra en una de las siguientes localizaciones: 1. 2. 3. 4. 5. Metfisis femoral distal Metfisis tibial superior Diafisis del radio Cuello femoral Metfisis tibial posterior

32.

La desinserccin del tendn del msc. supraespinoso suele ocurrir en las fracturas de : 39. 1. 2. 3. 4. 5. Cuello qco. del hmero. Troquiter. Troqun. Cuello anatmico del hmero. Luxacin anterior del hombro.

Habitualmente en un desprendimiento de retina concurren una serie de factores de riesgo entre los que no se encuentran: 1. 2. 3. 4. 5. Miopa. Edad. Afaquia. Hipermetropa. Degeneraciones perifricas.

33.

Las fracturas ms frecuentes causadas por metstasis seas se localizan en : 1. Pala iliaca. 2. Cuello de hmero. 40.

La manifestacin MAS frecuente del glaucoma congnito es:

-4-

Centro de Estudios Aula Neptuno - GRANADA


www.aulaneptuno.com

Ver.: 10/2001 47. Para que la prueba de la T de Studente pueda aplicarse es necesario que la variable siga una distribucin normal (condicin que puede obviarse si el nmero de sujetos es superior a 30) y que las varianzas sean similares en ambos grupos, hecho que se conoce como: 1. 2. 3. 4. 5. 48. Homoscedasticidad. Isotermia. Isocronia. Isodifasismo. Homocistinuria.

1. 2. 3. 4. 5. 41.

Lagrimeo. Fotofobia. Estrabismo. Excavacin de la papila. Megalocrnea.

La blefaritis marginal o inflamacin de los bordes palpebrales se debe frecuentemente, en sus formas crnicas y agudas a infeccin por: 1. 2. 3. 4. 5. Bacilo xerosis. Bacilo de Koch. Chlamydia trachomatis. Staphiloccoco. Diplococcus gonorheae.

En la prueba de Mann Whitney o suma de los rangos de Wilcoxon, el parmetro que realmente se somete a contraste es: 1. 2. 3. 4. 5. La media artimtica. La media geomtrica. La mediana. La desviacin media. La moda.

42.

El msculo oblicuo superior tiene una inervacin a cargo de : 1. 2. 3. 4. 5. MOC. N. Troclear o Pattico. Simptico. N. Facial. MOE.

49.

Cul de las siguientes medidas de localizacin de una distribucin de frecuencias puede tomar ms de un valor? 1. 2. 3. 4. 5. La mediana. La moda. El percentil 10. La media aritmtica. El tercer cuartil.

43.

Una de las siguientes entidades NO se considera una verdadera urgencia ocular sobre las que habra que actuar lo antes posible sin demora : 1. 2. 3. 4. 5. Quemadura qumica de la crnea. OACR. Celulitis orbitaria. Ulcera corneal. Neuritis ptica.

50.

44.

La altura de los individuos de una poblacin sigue una distribucin normal de media 165 cm y disviacin tpica de 15 cm. Qu porcentaje de individuos, aproximadamente, medirn menos de 180 cm?: 1. 2. 3. 4. 5. 68%. 34%. 97,5%. 84%. 88%.

Al realizar una comparacin entre dos tratamientos para una cierta enfermedad mediante una t de Student, qu determinacin sera til para complementar los datos proporcionados por el valor p? 1. 2. 3. 4. 5. El anlisis de la varianza. El coeficiente de variacin. El coeficiente de correlacin de Pearson. El clculo del intervalo de confianza. El valor del estadstico chi cuadrado.

51.

La probabilidad de que en la distribucin normal 50, 5 un valor se site por debajo de 62.5 es igual a la probabilidad de que en la normal tpica un valor se sita por debajo de: 1. 2. 3. 4. 5. 2.5. -1.5. 2. 2.25. 1.75.

45.

El mtodo de medida que se ha utilizado para las determinaciones del anterior ejemplo tiene un coeficiente de variacin de: 1. 2. 3. 4. 5. 15/180. 15/165. 165/180. 165/15. 180/15.

52.

46.

Qu afirmacin es cierta en la distribucin binomial?: 1. Puede tomar cualquier valor desde cero hasta infinito. 2. Es una variable cuantitativa continua. 3. La probabilidad de verificar la caracterstica es inconstante en los individuos observados. 4. No puede aproximarse a la normal. 5. La probabilidad de no verificar la caracterstica es: (1-p).

En un test bilateral de contraste de hiptesis afirmo que, de existir una diferencia significativa a favor de uno de los frmacos estudiados, el test tena un 70% de probabilidades de detectarlo. Cul es cierta? 1. El test no es significativo al ser p de 0,3. 2. Al ser bilateral el test no sern considerados significativos todos los resultados que indiquen diferencia entre ambos tratamientos. 3. Las decisiones por H1 van acompaadas de un error del 30%. 4. La potencia del test es del 70%. 5. En los test bilaterales el error alfa debe ser del 10%.

-5-

Centro de Estudios Aula Neptuno - GRANADA


www.aulaneptuno.com

Ver.: 10/2001 58. Tras ser sometido a laparotoma por diverticulitis complicada un enfermo presenta dolores violentos epigstricos, vmitos copiosos sin esfuerzo por parte del paciente, distensin abdominal con abombamiento de hipocondrio izquierdo y epigastrio y disminucin de ruidos peristlticos. No hay defensa muscular. En una raidiografa simple de abdomen observamos una gran dilatacin de la cavidad gstrica con un largo nivel hidroareo. EL diagnstico es: 1. 2. 3. 4. 5. 59. Dilatacin aguda del estmago. Vlvulo gstrico. Estenosis hipertrfica del ploro. Atresia duodenal. Bezoar gstrico.

53.

Para comparar la eficacia relativa de dos frmacos para el tratamiento de la cefalea elijo dos grupos de 20 pacientes cada uno y observo cuntos experimentan mejora del dolor tras 24 horas de administracin. Una afirmacin es falsa en este caso: 1. La prueba estadstica adecuada para realizar la comparacin es la chi cuadrado. 2. El estudio deber ser completado por una comparacin de cada uno de los frmacos contra el placebo. 3. Una p menor del 1% indicar que existen esas probabilidades de que la diferencia encontrada sea debida al azar. 4. El test realizado es bilateral o de dos colas. 5. Si me informan del intervalo de confianza para la proporcin que refleja la diferencia entre uno y otro tratamiento, este resultado ser significativo si el intervalo excluye el valor cero.

El tratamiento de eleccin de los adenocarcinomas de la cabeza del pncreas es: 1. 2. 3. 4. 5. Pancreatectoma total. Duodenopancreatectoma ceflica. Colecistoyeyunostoma. Coledocoyeyunostoma. Duodenopancreatectoma ceflica.

54.

Paciente de 18 aos de edad, que ingresa en el hospital por accidente de trfico sufrido ocho horas antes. En la urgencia se aprecia roce pericrdico, ausencia de pulso axilar izquierdo, paraparesia (dice haber notado parapleja) y, en la radiografa de trax se aprecia ensanchamiento mediastnico y fractura de la primera costilla. El paciente impresiona de grave. El diagnstico ms probable es: 1. 2. 3. 4. 5. Infarto agudo de miocardio. Contusin miocrdica. Embolismo de pulmn. Diseccin traumtica de la aorta. Endocarditis bacteriana.

60.

Si en un politraumatizado existen todas estas lesiones, cul trataras en primer lugar? 1. 2. 3. 4. 5. Rotura vesical extraperitoneal. Fractura de tres costillas. Fractura abierta de tibia y peron. Diseccin traumtica de la aorta. Rotura renal parcial.

61.

EN el tratamiento de las heridas infectadas, la sutura secundaria se realiza habitualmente: 1. 2. 3. 4. 5. A las 48 horas de producirse la herida. A los 15-20 das. A los 4-5 das. A partir de las tres semanas. Si la primaria no se ha cosido bien.

55.

Cul de las siguientes afirmaciones no es correcta con respecto al hematoma subdural crnico? 1. Puede simular el cuadro clnico de un tumor cerebral. 2. La ausencia de antecedentes de traumatismo craneal previo en la anamnesis permite descartarlo. 3. El diagnstico se realiza habitualmente con la TAC craneal. 4. Puede presentarse como demencia. 5. La mayor parte se evacuan quirrgicamente.

62.

Una de las siguientes caractersticas no corresponde a la prpura trombtica trombocitopnica: 1. Depsito microvascular de trombos hialinos con fibrina. 2. Marcada reaccin inflamatoria en la pared vascular. 3. Anemia hemoltica microangioptica Coombs negativa. 4. Nivel de conciencia fluctuante. 5. Normocomplementemia.

56.

La compliacin tarda ms frecuente de un traumatismo grave de la uretra posterior es : 1. 2. 3. 4. 5. Absceso periuretral. Fstula urinaria. Impotencia. Estenosis de la uretra. Hematocele.

63.

El tratamiento de eleccin en la anterior enfermedad es: 1. 2. 3. 4. 5. Exanguinotransfusin o plasmafresis. Tratamiento antiplaquetario. Corticoides o inmunosupresores. Esplenectoma y corticoides. Heparina y transfusin de plaquetas.

57.

Cul es la localizacin ms frec. de los vlvulos colnicos ? : 1. 2. 3. 4. 5. Sigma. Colon ascendente. Recto. Colon transverso. Ciego.

64.

En un nio con prpura petequial, artralgias y hematuria, el estudio de la hemostasia refleja: Plaquetas: 300.000/ml; T.Duke: 4'; TTPA: 39"; T.Protrombina: 13"; agregacin con

-6-

Centro de Estudios Aula Neptuno - GRANADA


www.aulaneptuno.com

Ver.: 10/2001 en las clulas precursoras de mdula sea. 2. La aparicin de manifestaciones de hiperviscosidad. 3. La intensa trombocitosis. 4. El hallazgo de alteraciones inmunolgicas como ANA, hipocomplementemia, etc. 5. La reduccin de la actividad de la fosfatasa alcalina leucocitaria. 70. En una de las siguientes circunstancias aumentar la afinidad de la hemoglobina por el oxgeno, por lo que la curva de saturacin de aqulla se desplazar hacia la izquierda y se dificultar la cesin de oxgeno a los tejidos. 1. En un paciente con hiperfosfatemia secundaria a hipoparatiroidismo. 2. En un paciente con proceso febril agudo. 3. A nivel del alveolo pulmonar. 4. En un paciente con depresin respiratoria por sobredosis de opiceos. 5. A nivel del lecho capilar intestinal. 71. Indica la falsa sobre el diagnstico diferencial de las siguientes anemias: 1. En la anemia de las enfermedades crnicas la ferritina est alta y la TIBC est descendida, al contrario de lo que sucede en la ferropenia. 2. En la ferropenia el ndice de distribucin eritrocitaria es mayor que el observado en las talasemias. 3. En la esferocitosis la resistencia osmtica eritrocitaria est claramente descendida. Sucede lo contrario en las talasemias. 4. En las anemias megaloblsticas por dficit de vitamina B12 existe clnica neurolgica, no observable en las megaloblsticas por folicopenia. 5. EN las anemias sideroblsticas el ndice de saturacin de la transferrina est descendido, al igual que en las anemias de las enfermedades crnicas. 72. Cul de las siguientes formas de embarazo ctopico es ms frecuente? 1. 2. 3. 4. 5. 73. Embarazo tubrico ampular Embarazo tubrico itsmico Embarazo tubrico intersticial Embarazo ovrico Embarazo abdominal

RCO normal; disolucin del cogulo en urea 5M: normal. Dgto.ms probable: 1. 2. 3. 4. 5. 65. Werlhof. Dficit del factor XIII. Von Willebrand. Hemofilia A. Schonlein-Henoch.

Respecto a la hemoglobinuria paroxstica nocturna, es falso: 1. Un paciente con HPN puede no presentar nunca hemoglobinuria macroscpica. 2. Se produce por una alteracin de la membrana, adquirida a nivel de las clulas precursoras. 3. Hay una especial sensibillidad a la accin del complemento,por dficit de protenas reguladoras del mismo. 4. Suele haber dficit de hierro por la hemosiderinuria. 5. Puede haber fenmenos trombticos, por lo que se recomienda el uso de heparina durante las crisis hemolticas.

66.

Uno de los siguientes datos no ir a favor del diagnstico de hemlisis ante un paciente cansado y con ictericia: 1. Elevacin de la bilirrubina indirecta y la LDH en suero. 2. Bilirrubinuria. 3. Haptoglobina srica indetectable. 4. Recuento de reticulocitos aumentado. 5. Hemoglobinuria.

67.

Cul de las siguientes afirmaciones sobre la esferocitosis hereditaria es correcta? 1. Puede pasar desapercibida hasta la vida adulta. 2. En las personas sanas, los esferocitos representan el 10% o ms de los hemates de sangre perifrica. 3. Los esferocitos de la esferocitosis hereditaria se pueden distinguir de los de la anemia hemoltica autoinmune mediante la prueba de la fragilidad osmtica. 4. Generalmente la esplenectoma elimina los esferocitos de la sangre perifrica. 5. Raramente se acompaa de esplenomegalia.

68.

Las clulas de un nio con leucemia aguda expresan TdT, CALLA, CD34 y CD19. No tienen Igs.de membrana, ni antgenos de clulas T (CD2, CD7...). Debe clasificarse dicha leucemia como: 1. 2. 3. 4. 5. Nula. Comn. Linfoblstica de estirpe B. Mieloblstica sin maduracin. L3.

El plano de HODGE que coincide con el borde inferior de la snfisis pbica corresponde al: 1. 2. 3. 4. 5. Primer plano Segundo plano Tercer plano Cuarto plano Quinto plano

74.

El test de Silverman y Anderson incluye lo siguientes excepto: 1. 2. 3. 4. 5. Tono muscular Movimientos toraco-abdominales Retraccin xifoidea Aleteo nasal Quejido respiratorio

69.

Dentro de los sndromes mieloproliferativos crnicos, cul de las siguientes caractersticas es ms especfica de la mielofibrosis idioptica? 1. La presencia de alteraciones cromosmicas

-7-

Centro de Estudios Aula Neptuno - GRANADA


www.aulaneptuno.com

Ver.: 10/2001 do por una ascitis masiva e intensos edemas que dejan fvea en miembros inferiores y regin sacra. Los datos de laboratorio al ser ingresado son: Na 135; K 3.0; Cl 95; HCO3 28. BUN 15 mg/dl; creatinina 2.0 mg/dl. En el primer da de hospitalizacin la diuresis es de 400 ml con una concentracin de sodio de 6 mmol/L y una osmolaridad de 638 mosm/kg. Cul de las siguientes medidas es probable que sea la ms beneficiosa para tratar el estado de retencin de sodio descrito? 1. Furosemida oral, 120 mg dos veces al da. 2. Furosemida intravenosa 120 mg dos veces al da. 3. Manitol intravenoso 25 mg dos veces al da. 4. Paracentesis abdominal para drenar dos litros de lquido diariamente. 5. Reposo en cama y dieta pobre en sodio. 83. Nia de 10 aos, llevada a una servicio de urgencias, desde el que ingresa en el servicio de Nefrologa por HTA, edemas, proteinuria y hematuria macroscpica. El C3 srico est descendido, y el C3NeF es positivo. En la biopsia renal hay depsitos en la membrana basal glomerular, que engruesan la misma. Se encuentran asmismo algunos ndulos mesangiales PAS (+). El dgto.es: 1. 2. 3. 4. 5. 84. GN membranosa. GN mesangiocapilar tipo II. Hialinosis focal y segmentaria. Nefropata mesangial IgA. GN postestreptoccica.

75.

Cul es la tcnica de eleccin para el dgto.de la placenta previa? 1. 2. 3. 4. 5. Placentografa indirecta. Placentografa directa. Termografa. Localizacin isotpica. Ecografa.

76.

La localizacin ms frecuente del cncer de mama es: 1. 2. 3. 4. 5. Cuadrante supero interno. Cuadrante supero externo. Cuadrante infero-interno. Retroareolar. Cuadrante infero-externo.

77.

Entre el espectro de lesiones premalignas de la mama, cul se asociara a un mayor riesgo de padecer cncer? 1. 2. 3. 4. 5. Ectasia ductal. Quistes. Hiperplasia epitelial. Adenosis. Fibroadenoma.

78.

Se denomina cloasma a la aparicin de: 1. 2. 3. 4. 5. Estras en el abdomen. Pigmentacin en la cara. Araas vasculares. Eritema palmar. Hemangiomas.

79.

El aumento asimtrico del tero, que adems es muy blando, recibe el nombre de: 1. 2. 3. 4. 5. Signo de Noble-Budin. Signo de Piskacek. Signo de Hegar. Signo de Chadwik. Signo de Kustner. 85.

Una de las siguientes asociaciones causa-efecto txico renal es falsa: 1. 2. 3. 4. Litio - Diabetes inspida nefrognica. Plomo - Tubulopata proximal. Aspirina - Diabetes inspida nefrognica. Ciclosporina - Vasoconstriccin de la arteriola aferente. 5. Aminoglucsidos - Necrosis papilar. En un paciente con Sndrome de Liddle no encontraremos: 1. 2. 3. 4. 5. Hipertensin arterial. Hipopotasemia. Niveles elevados de aldosterona en plasma. Alcalosis metablica. Reabsorcin aumentada de sodio en los tbulos distales.

80.

Cul de las siguientes porciones del sistema tubular tiene ribete en cepillo y abundantes microvellosidades? 1. 2. 3. 4. 5. T.colector. Asa de Henle. T.contorneado distal. Pars recta del t.distal. T.contorneado proximal.

81.

Entre los tratamientos empleados en la insuficiencia renal aguda para impedir la progresin de las lesiones celulares renales y facilitar su reparacin se emplean todos los tratamientos que siguen, excepto uno: 1. 2. 3. 4. 5. Inhibidores de la xantina oxidasa. Inhibidores de la ECA. Antagonistas del calcio. Quelantes de radicales libres de oxgeno. Perfusiones de soluciones intracelulares (xido de magnesio, nucletidos y aminocidos).

86.

Ante un cuadro de dolor en el flanco, hematuria, piuria, aumento de la proteinuria y deterioro de la funcin renal en un paciente con nefropata membranosa, lo ms probable es que se trate de: 1. 2. 3. 4. 5. Trombosis de la arteria renal. Embolia de la arteria renal. Trombosis de la vena renal. Proliferacin extracapilar difusa glomerular. Pielonefritis aguda.

87. 82. Un hombre de 35 aos con antecedentes de alcoholismo y cirrosis heptica es hospitaliza-

Nio en estudio por dolores seos y excesiva facilidad para las fracturas, con retraso del crecimiento, poliuria y polidipsia. En la anal-

-8-

Centro de Estudios Aula Neptuno - GRANADA


www.aulaneptuno.com

Ver.: 10/2001 cambios mnimos y recidivas frecuentes tras tratamiento corticoideo, que han sido tratados con ciclos de ciclofosfamida o clorambucil permanecen libres de enfermedad a los 5 aos. 3. En los nios con enfermedad de cambios mnimos la frecuencia de recidivas aumenta con la edad. 4. Tanto entre los nios como entre los adultos, la frecuencia de recidivas se sita en torno al 50-65% durante la retirada o la interrupcin del tratamiento. 5. Una pauta que se recomienda frecuentemente como tratamiento inicial es la prednisona diaria durante cuatro semanas seguida por prednisona en das alternos cuatro semanas ms. Las recidivas se pueden tratar con la misma pauta pero con retirada ms gradual y dosis bajas de mantenimiento durante tres a seis meses. 93. EN un paciente con nefropata diabtica en el que la funcin renal es an aceptable (aclaramiento de creatinina del 60%), encontrar una acidosis metablica hiperclormica con anin GAP dentro de los lmites de la normalidad debe hacer pensar en: 1. 2. 3. 4. 5. 94. Hipoaldosteronismo hiporreninmico. Acidosis tubular proximal. Cetoacidosis diabtica. Acidosis del fracaso renal leve. intoxicacin por biguanidas.

tica se encuentra una acidosis metablica con pH de 7,098 ,bicarbonato de 18 mEq/L y potasio de 2,9 mEq/L. Se demuestra que el rin elimina el 19% del bicarbonato filtrado. Cul de estas causas es la que menos probablemente habr originado este trastorno? 1. 2. 3. 4. 5. 88. Tratamiento con acetazolamida. Enf.de Wilson. Cistinosis. Diabetes mellitus. S.de Fanconi.

El tratamiento de eleccin en un tumor vesical superficial es: 1. 2. 3. 4. 5. Instilaciones con antimitticos. Radioterapia externa. Reseccin transuretral. Quimioterapia. Cistectoma parcial.

89.

Las hemorragias petequiales puntiformes que aparecen en toda la extensin vesical despus de la hidrodistensin son caractersticas de: 1. 2. 3. 4. 5. Cistopata glandular. Cistitis eosinfila. Cistitis de clulas claras. Cistopata cervicotrigonal. Cistitis intersticial.

90.

Un hombre de 55 aos se somete a una pielografa intravenosa como parte del estudio para su hipertensin arterial. En el rin izquierdo se aprecia una masa solitaria radiotransparente de 3 cm.de dimetro; por lo dems, el estudio es normal. El hombre no se quehja de sntomas de vas urinarias y el examen del sedimento urinario est dentro de lmites normales. Cul de los siguientes estudios debera realizarse a continuacin? 1. Repetir la PIV pasados 6 meses. 2. Recogida de la primera orina de la maana para efectuar citologa. 3. Arteriografa renal selectiva. 4. Ultrasonografa renal y, si es preciso, aspiracin con aguja de la masa. 5. TAC con contraste de refuerzo del rin izquierdo.

Sntomas vestibulares espontneos y provocados, as como una hipoacusia de percepcin, unidos a parlisis facial homolateral, parlisis de los pares IX y X y ataxia cerebelosa, son caractersticos de: 1. 2. 3. 4. 5. Neuronitis vestibular. Insuficiencia vertebro-basilar. Parlisis bulbar. Tumor del ngulo ponto-cerebeloso. Enfermedad de Menire.

95.

En qu proceso debe pensarse con mayor probabilidad cuando un recin nacido al mamar por primera vez se pone ciantico y disneico? 1. 2. 3. 4. 5. Plipo nasal congnito. Fibroma nasofarngeo. Atresia coanal. Hiperplasia de amgdala palatina. Hiperplasia de amgdala farngea.

91.

Un paciente en estudio por litiasis renal, presnta en el sedimento de orina cristales hexagonales, siendo positivo el test del nitroprusiato. Qu tratamiento emplearas en l? 1. 2. 3. 4. 5. Alopurinol. Acidificar la orina. Tiacidas. Acetohidroxmico. Alcalinizar la orina. 96.

92.

Una es falsa sobre el tratamiento del sndrome nefrtico por lesiones mnimas: 1. El tratamiento inicial con metilprednisolona a altas dosis puede acelerar la aparicin de la remisin, pero tambin aumenta el porcentaje de recidivas. 2. La mitad de los pacientes con enfermedad de

En un paciente con clnica de otitis media infecciosa aparece un aumento de la sintomatologa con presencia de fiebre, leucocitosis, aceleracin de la VSG, gran otalgia y supuracin, as como alteracin del estado general. A la inspeccin se observa una tumefaccin retroauricular caliente y dolorosa, y que motiva separacin del pabelln auricular. Cul es el diagnstico ms probable? 1. 2. 3. 4. Tromboflebitis del seno lateral. Mastoiditis. Petrositis. Enfermedad de Lemire.

-9-

Centro de Estudios Aula Neptuno - GRANADA


www.aulaneptuno.com

Ver.: 10/2001 zar en esta mujer debe ser: 1. Colocacin de un marcapaso ventricular de demanda permanente. 2. Probar con un marcapaso transvenoso temporal. 3. Iniciar tratamiento con isoproterenol sublingual. 4. Monitorizacin continua con Holter durante 24 horas. 5. Pruebas de tolerancia al ejercicio. 104. Paciente de 47 aos, ejecutivo de "Arthur Andersen", fumador importante que acude a un servicio de urgencias por dolor precordial que le ha despertado durante el reposo nocturno de 3 horas de evolucin. El dolor lo define como opresin con sensacin de falta de aire, sudoracin fria y nuseas; se irradia hacia mandbula, hombro izquierdo y regin interescapular. En el ECG se onserva elevacin del S-T en II, III y aVF as como descenso del S-T en I y aVL. No existen ondas Q. En la analtica de sangre la CPK es de 127 U/L y la LDH de 372 U/L. La CPK-MB es de 11 U/L. La actitud CORRECTA sera: 1. Traslado a UVI y actitud conservadora. 2. Registro de precordiales derechos (V4R) y traslado a UVI. 3. Administracin de NTG sl y una vez controlado el dolor, realizacin de test de hiperventilacin. 4. Realizacin de manometra esofgica. 5. Coronariografa urgente. 105. Entre las caractersticas de la miocardiopata producida por la amiloidosis slo se encuentra una de las que se citan a continuacin: 1. Salvo casos de extrema necesidad, est contraindicado el empleo de anticoagulantes orales por el alto riesgo de complicaciones hemorrgicas. 2. Existe una elevacin de presiones diastlicas en el corazn izquierdo, siendo normales las de las cmaras derechas. 3. La principal alteracin es la reduccin de la fraccin de eyeccin. 4. Suele cursar con hipertensin arterial rebelde al tratamiento. 5. El tratamiento de eleccin se hace con digital, diurtico y vasodilatadores arteriales. 106. En el diagnstico diferencial entre estenosis e insuficiencia mitral crnicas, slo uno de estos datos es tpico de la insuficiencia: 1. 2. 3. 4. Ms frecuente en hombres. Mayor frecuencia de embolias. Curso episdico. La decisin quirrgica se basa en la sintomatologa del enfermo. 5. Hemoptisis de repeticin. 107. En una de las siguientes cardiopatas congnitas con shunt izquierda-derecha la saturacin de oxgeno es normal en ambas cavidades derechas: 1. Aneurisma del seno del Valsalva.

5. Laberintitis. 97. Uno de los siguientes signos clnicos o exploratorios no es propio de la otoesclerosis: 1. 2. 3. 4. 5. 98. Signo de la fstula. Ausencia de reflejo estapedial. Signo de Schwartze. Paracusia de Willis. Muesca de Carhart en la frecuencia de 2000 Hz en la va sea.

Cul de las siguientes afirmaciones es cierta respecto al nistagmo del vrtigo posicional paroxstico? 1. Puede provocarse. 2. Aparece de forma inmediata al posicionar la cabeza del paciente, sin latencia. 3. Es pendular. 4. No se acompaa de sensacin de giro de objetos. 5. Aparece tantas veces consecutivas cuantas provoquemos el cambio posicional (Signo del "nistagmo de los 100 das").

99.

En el tratamiento de la angina variante no se usa: 1. 2. 3. 4. 5. Molsidamina. Nitroglicerina s.l. Felodipino. Atenolol. Nifedipino.

100. Si un paciente de 49 aos con angina inestable tiene en la coronariografa una lesin significativa en el tronco comn de la arteria coronaria izquierda, el tratamiento ms recomendable ser: 1. 2. 3. 4. 5. Trasplante cardiaco. ACTP. Aterotoma de Simpson. Nitratos y beta bloqueantes. By pass aorto coronario.

101. Cul es la causa ms comn de embolias de origen cardiaco? 1. 2. 3. 4. 5. La diseccin de aorta. La endocarditis protsica precoz. La miocardiopata dilatada. La endocarditis protsica tarda. La estenosis mitral con fibrilacin auricular.

102. Una de las siguientes asociaciones referente al pulso arterial no es cierta: 1. Pulso pequeo o parvus: shock. 2. Pulso saltn: Estados circulatorios hiperquinticos. 3. Pulso dcroto: Estenosis artica. 4. Pulso grande o magnus: Insuficiencia artica. 5. Pulso bisferiens: Miocardiopata hipertrfica. 103. Una mujer de 79 aos tiene, diariamente, episodios de mareo. Un trazado para ver el ritmo demuestra bradicardia sinusal de 52 lpm con pausas sinusales de 2,5 seg.que no producen sntomas. El siguiente paso a reali-

- 10 -

Centro de Estudios Aula Neptuno - GRANADA


www.aulaneptuno.com

Ver.: 10/2001 co utilizados en la fase aguda del IAM. 4. Los diurticos son inocuos durante el embarazo. 5. La administracin de potasio produce un descenso transitorio de la tensin arterial. 112. Sobre la tromboangetis obliterante o Enfermedad de Buerger no es cierto que: 1. Las manifestaciones iniciales ms habituales son la claudicacin y el dolor en reposo de los miembros inferiores. 2. La lesin anatomo patolgica consiste en la existencia de trombosis microvascular, no hallndose cambios inflamatorios en la pared vascular. 3. El nico tratamiento que se ha mostrado eficaz es la abstencin de fumar. 4. La enfermedad afecta de forma preferente a menores de 40 aos muy fumadores. 5. Los pulsos perifricos suelen ser dbiles, mientras que los proximales se hallan conservados. 113. Un hombre de 44 aos fue enviado para tto. de su HTA. Se quejaba de fatigabilidad fcil y disnea de esfuerzo leve, pero neg otros sntomas cardiacos. Exploracin fsica : pulso regular a 90 ppm, TA 170/100 mm Hg, Cardiaca : Impulso apical vigoroso en el quinto espacio intercostala 2 cm a la izda de la linea medioclavicular; soplo sistlico eyectivo II/VI a la izda. del esternn. Extremidades: pulsos femorales presentes pero dbiles. Rx de trax: muescas costales bilaterales, y botn artico pequeo. Cul es el hallazgo ms frecuente de los siguientes que puedes encontrar en la ecocardiografa? : 1. 2. 3. 4. 5. CIA tipo ostium secumdum. Ductus arterioso persistente. Vlvula artica bicspide. Prolapso de vlvula mitral Anomala de Ebstein.

2. 3. 4. 5.

Comunicacin interauricular. Persistencia del ductus. Comunicacin interventricular. Canal A-V.

108. El tratamiento con diurticos ser til en un paciente que padezca: 1. 2. 3. 4. 5. Miocardiopata restrictiva. Miocardiopata dilatada. Pericarditis constrictiva. Taponamiento cardiaco. Infarto del ventrculo derecho.

109. Varn de 57 aos que acude a urgencias porque estando trabajando en su jardn sufre bruscamente prdida de la visin en campo superior izquierdo junto con prdida de fuerza en extremidad inferior izquierda, seguido de dolor brusco con frialdad y cianosis de extremidad superior derecha. Antecedentes: Hiperglucemia en tratamiento diettico. Fumador de 10 cigarrillos/da. Diagnosticado de valvulopata mitral en fibrilacin auricular, tratado con digitlicos y amiodarona. Disnea ante esfuerzos mayores que los realizados en su vida habitual. Exploracin: Pulsos carotdeos arrtmicos y simtricos. Ausencia de pulsos cubital y radial derechos. TA en brazo izquierdo: 170/100. Auscultacin cardiaca: Refuerzo de S1 mitral, chasquido de apertura cerca del segundo ruido y retumbo largo. Ausc.pulmonar normal. Abdomen blando. No edemas. Exploracin neurolgica: Cuadrantanopsia superior izquierda. Claudicacin de miembro superior derecho. Ligero dficit motor en extremidad inferior izquierda. Sensibilidad y reflejos conservados a todos los niveles. ECG: Arritmia completa por fibrilacin auricular con RVM a 80 lpm. Eje elctrico a 90 en el plano frontal. Rx.de trax: Cardiomegalia I/IV con crecimiento de AI y patrn de redistribucin del flujo pulmonar. En el diagnstico del paciente pondras todos los siguientes, excepto uno: 1. 2. 3. 4. 5. Insuficiencia cardiaca congestiva. Estenosis mitral pura en grado funcional I. Fibrilacin auricular. ACV temporo-occipital derecho. Embolia en extremidad superior derecha.

114. Seale la respuesta INCORRECTA en relacin al Sd. de Eisenmenger: 1. Uno de los sntomas puede ser la angina de esfuerzo. 2. Es caracterstica la presencia de Policitemia. 3. Pueden producirse sncopes de esfuerzo. 4. La Rx de trax muestra la correccin del defecto que ha ocasionado el sd. 5. El tto. consiste en la correccin del defecto que ha ocasionado el sndrome. 115. Cul de las siguientes representa la principal limitacin del tratamiento con diurticos de asa en pacientes con cor pulmonale? 1. Que, indirectamente, pueden provocar aumento de las resistencias pulmonares. 2. Que aumentan la postcarga del ventrculo izquierdo. 3. Que provocan hipercalcemia. 4. Que producen un aumento medio del valor hematocrito de un 5% en pacientes con EPOC tipo bronquitis crnica. 5. Que provocan alcalosis metablica.

110. Cmo procedera Vd ante un sujeto que desarrolla rapidamente aumento de la presin venosa, hipotensin arterial y pulso arterial paradjico? 1. 2. 3. 4. 5. Administracin de digital. Administracin de quinidina. Perfusin de expansores del plasma. Pericardiocentesis de urgencia. Colocacin de un marcapasos.

111. Es falso que: 1. El nitroprusiato i.v. es el tratamiento de eleccin en la diseccin artica. 2. La nitroglicerina i.v. est contraindicada en el infarto de miocardio inferior con afectacin del ventrculo derecho. 3. Los beta bloqueantes i.v. mejoran el pronsti-

- 11 -

Centro de Estudios Aula Neptuno - GRANADA


www.aulaneptuno.com

Ver.: 10/2001 1. Ocurre aproximadamente en un 10 % de los pacientes. 2. Suele cursar con adenopatas y ndulos pulmonares. 3. Un 10% pueden desarrollar un linfoma linfoctico. 4. El tratamiento con citostticos puede favorecer su transicin a linfoma verdadero. 5. Requiere tratamiento hospitalario urgente. 120. Qu manifestacin no se suele observar en el Behet? 1. 2. 3. 4. 5. Ulceras orales. Eritema nodoso. Artritis erosiva. Tromboflebitis. S.de vena cava superior.

116. Qu afirmacin es falsa en relacin a la prueba de esfuerzo? 1. Las pruebas de esfuerzo modificadas (limitadas por la frecuencia cardiaca ms que por los sntomas) pueden realizarse con seguridad incluso siete das despus del infarto de miocardio. 2. La mortalidad de la prueba se cifra en uno por cada diez mil ergometras. 3. Si la presin arterial disminuye durante la prueba es signo de mal pronstico, ya que puede reflejar una disfuncin ventricular izquierda inducida por la isquemia. 4. La prueba de esfuerzo puede completarse con la administracin intravenosa de un istopo radiactivo para la evaluacin de la perfusin miocrdica regional. 5. la ecocardiografa no tiene utilidad en el diagnstico de la cardiopata isqumica. 117. El cateterismo del lado izquierdo del corazn y la coronariografa estn indicados especialmente en los pacientes con Estenosis Artica y los siguientes supuestos, excepto uno. Selalo: 1. Pacientes con cardiopata multivalvular en los que antes de planificar el tratamiento quirrgico hay que definir la importancia de cada lesin valvular. 2. Paciente con sospecha clnica de Estenosis Artica pero con Ecocardiografa normal. 3. Pacientes jvenes asintomticos con EA congnita no calcificada para definir la gravedad de la obstruccin, ya que si es grave puede estar indicada la intervencin quirrgica. 4. Pacientes en los que se sospecha que la obstruccin grave no est en la vlvula artica sino en las regiones sub o supravalvulares. 5. Pacientes con signos clnicos de EA e isquemia miocrdica en los que se sospecha cardiopata coronaria. 118. Sobre las consideraciones especiales en el tratamiento de la Hipertensin Arterial es falso que: 1. La disminucin de la presin arterial en los hipertensos con deterioro de la funcin renal se acompaa inicialmente de un aumento de la creatinina srica. 2. En la mujer embarazada los inhibidores de la ECA constituyen el tratamiento de primera lnea, por sus escasos efectos metablicos y su inocuidad sobre el feto. 3. Los inhibidores de la ECA son particularmente tiles en los diabticos, por su ausencia de efectos adversos sobre el metabolismo de la glucosa y por dificultar el desarrollo de nefropata diabtica. 4. Se ha demostrado una reduccin de la mortalidad en ancianos con HTA tratados con dosis bajas de antihipertensores. 5. En cardipatas en tratamiento con digitlicos hay que administrar con precaucin las tiacidas e investigar las posibles disminuciones de los niveles sricos de potasio. 119. Respecto al seudolinfoma en el Sndrome de Sjgren, seala la incorrecta:

121. Respecto al embarazo en las mujeres con lupus, seala la correcta: 1. El tratamiento de eleccin, si se produjera un brote de actividad cutnea, seran los antipaldicos. 2. Se debe recomendar la anticoncepcin de por vida. 3. El lupus se suele exacerbar en el postparto. 4. Si se asocia anticoagulante lpico, el tratamiento de eleccin son los dicumarnicos. 5. De utilizar esteroides, se emplearn dexametasona y betametasona, que no atraviesan barrera placentaria. 122. Mujer de 48 aos, sin antecedentes de inters, que consulta con su mdico por dolor intenso e impotencia funcional leve a nivel de las articulaciones interfalngicas distales. A la exploracin, stas aparecen tumefactas, siendo el resto de la exploracin absolutamente normal. Cul de las siguientes afirmaciones es cierta sobre este cuadro clnico? 1. Un lquido sinovial transparente, con viscosidad elevada y buen cogulo de mucina, confirmara el dgto. de artritis reumatoide. 2. El lupus no se considerara en el dgto. diferencial, ya que nunca ocasiona artritis de interfalngicas distales. 3. La artritis purulenta es la causa ms probable del cuadro descrito. 4. El hallazgo de ndulos en el dorso de los dedos orientara hacia la artrosis como diagnstico. 5. La artritis psorisica queda descartada al no existir lesiones cutneas, que siempre preceden a la artritis. 123. En una paciente que debuta con fenmeno de Raynaud y tumefaccin de los dedos, sospechas esclerodermia. Todos los siguientes datos, excepto uno, van a apoyar tu primer dgto. Cul es esa excepcin? 1. 2. 3. 4. 5. Disfagia y pirosis. Parlisis facial perifrica. Disnea y tos seca. Calcinosis periarticulares. HTA con aumento de la ARP.

- 12 -

Centro de Estudios Aula Neptuno - GRANADA


www.aulaneptuno.com

Ver.: 10/2001 4. Osteoartrosis generalizada. 5. Enf.de Paget con artrosis secundaria. 128. La comprobacin en una Rx.de columna, de una calcificacin del anillo fibroso del disco intervertebral, es muy sugerente de: 1. 2. 3. 4. 5. Artrosis. Artritis reumatoide. Mal de Pott. Espondilitis anquilosante. Artritis de Reiter.

124. Cul es le procedimiento diagnstico de eleccin ante una artritis aguda de la primera metatarsofalngica? 1. Radiografa comparativa de la articulacin y la contralateral. 2. Acido rico srico. 3. Obtencin de lquido sinovial, efectuar Gram y cultivo y visin con el microscopio de luz polarizada. 4. Iniciar tratamiento con colchicina y ver respuesta. 5. No es necesario ningn procedimiento diagnstico, la localizacin es especfica de artritis gotosa. 125. Respecto a la artritis por microcristales, seala la falsa: 1. Puede estar producida no slo por urato y pirofosfato, sino tambin por hidroxiapatita y oxalato clcico. 2. Pueden coexistir cristales de diferentes sustancias en la misma articulacin. 3. Pueden asociarse enfermedad por microcristales y otra enfermedad articular, como artrosis o traumatismo previo. 4. Todas pueden causar tanto artritis aguda como artropata crnica destructiva y periartritis. 5. Las caractersticas clnicas y radiolgicas permiten diferenciar la mayora de las veces el tipo de cristal causante. 126. Seala la afirmacin incorrecta respecto al diagnstico de la sarcoidosis: 1. La actividad srica del ECA est elevada durante la fase activa de la enfermedad pero no tiene valor diagnstico. 2. La gammagrafa pulmonar con galio tampoco es diagnstica, aunque sirve para valora la actividad de la enfermedad. 3. El diagnstico se basa en la demostracin de granulomas no caseificantes por biopsia en un paciente con alta sospecha clnicoradiolgica. 4. La prueba de Kveim tiene muy poco valor para el diagnstico. 5. En el lavado broncoalveolar hay un predominio de los LT colaboradores. 127. Varn de 74 aos, que acude a consulta por dolor e impotencia funcional a nivel de la cadera derecha. Entre sus antecedentes, destacan hiperuricemia e hipercolesterolemia. El estudio radiolgico de dicha articulacin revela la existencia de reas lticas, reas de mayor densidad sea, aumento del tamao seo y deformidad, con disminucin de la interlnea articular, esclerosis subcondral y osteofitosis. Se aprecian tambin alteraciones radiolgicas parecidas en crneo y columna. En la analtica, calcio, fsforo y PTH son normales. Hay elevacin de fosfatasa alcalina, osteocalcina, hidroxiprolinuria y fosfatasa cida resistente al tartrato. El dgto. ms probable es: 1. Cncer de prstata metastatizado. 2. Tricoleucemia. 3. Artropata gotosa.

129. En relacin a las manifestaciones sistmicas del LES, slo es CIERTA una de las siguientes respuestas: 1. No estn incluidas en los captulos correspondientes de los apuntes de Centro de Estudios @ula Neptuno. 2. El aumento de transaminasas sricas en pacientes con LES indica una afectacin heptica importante y no es de esperar que se normalice su nivel con el tratamiento. 3. La endocarditis de Libman Sachs produce en la mayora de los casos insuficiencia valvular que motiva su recambio. 4. La mayora de las infecciones en pacientes con LES se producen como consecuencia de la frecuente leucopenia que presentan estos enfermos. 5. Ante crisis de dolor abdominal espasmdico agudo, vmitos y diarrea hay que pensar en una vasculitis intestinal. 130. Una de estas manifestaciones no es esperable en un paciente con Granuloma de la Lnea Media: 1. Ndulos pulmonares cavitados. 2. Erosin de un vaso importante y hemorragia masiva. 3. Lesiones destructivas y mutilantes en la piel de la cara y ojos. 4. Disfagia y dificultad para hablar. 5. La progresin puede acelerarse por el tratamiento quirrgico de las zonas afectadas. 131. Una de las siguientes manifestaciones no es especfica de la forma difusa de la esclerodermia, y se asocia ms habitualmente a las formas limitadas: 1. 2. 3. 4. 5. Anticuerpos anti DNA topoisomerasa I. Hipertensin arterial maligna. Hipertensin arterial pulmonar. Neumatosis qustica intestinal. Miopata.

132. Paciente de 68 aos de edad con fibrilacin auricular, que presenta paresia de la pierna izquierda de inicio brusco, asociada a incontinencia urinaria. La TAC craneal realizada en las primeras 12 horas es normal. El diagnstico ms probable es: 1. Infarto en el territorio de la arteria cerebral media derecha. 2. Infarto en la arteria cerebral posterior derecha. 3. Infarto en la arteria cerebral media izquierda.

- 13 -

Centro de Estudios Aula Neptuno - GRANADA


www.aulaneptuno.com

Ver.: 10/2001 136. Paciente con SIDA desarrolla cuadro de paresia derecha, ataxia y disminucion del grado de consciencia. Se realizo RNM que mostro areas de desmielinizacion diseminadas. Cual los siguientes diagnosticos es mas probable; 1. 2. 3. 4. 5. Leucoencefalopatia multifocal progresiva. Toxoplasmosis. Encefalopatia VIH. Criptococosis cerebral. Tuberculosis cerebral

4. Infarto en la arteria cerebral anterior derecha. 5. Infarto en la arteria cerebral anterior izquierda. 133. El tratamiento inicial de las personas con hipertensin intracraneal comprender: 1. Beta-bloqueantes. 2. Difenilhidantona. 3. Ventilacin mecnica para lograr presiones altas en las vas respiratorias. 4. Deshidratacin con sustancias hiperosmolares. 5. Lquidos intravenosos. 134. A un paciente alcohlico que se queja de trastornos del equilibrio se le encuentra que camina ampliando la base de sustentacin y que oscila hacia delante y hacia atrs estando de pie. No puede mantener el equilibrio estando de pie y con los ojos cerrados o abiertos. No se aprecia ataxia de miembros ni nistagmus. Estos datos son compatibles con una o ms lesiones de: 1. 2. 3. 4. 5. Sistema vestibular. La lnea media del cerebelo. El lbulo temporal. La regin lateral del cerebelo. La corteza frontal izquierda

137. En la evolucin y pronstico de un brote agudo monosintomtico de esclerosis mltiple, es verdad todo lo que sigue, excepto: 1. El pronstico es ms favorable si los sntomas son de una afectacin aislada de un par craneal, o exclusivamente de carcter sensitivo. 2. El pronstico es peor si hay bandas oligoclonales de IgG e IgM. 3. Son factores de pronstico menos malo el inicio del primer brote antes de los 25 aos y la remisin completa de los sntomas. 4. El pronstico descansa siempre en los datos que van surgiendo de los estadios evolutivos de la RMN. 5. El estudio seriado de PE visuales, auditivos y somatosensoriales tiene un valor predictivo en la evolucin. 138. Un nio de 9 aos presenta rigidez muscular en las extremidades, parlisis seudobulbar y distonas. Presenta tambin signos de deterioro intelectual. Hay una hiperpigmentacin de la sustancia negra, el ncleo rojo y el globo plido. Qu tratamiento mejorar la rigidez y las distonas? 1. 2. 3. 4. 5. Desferrioxamina. L-Dopa. Sulfato ferroso oral. D-penicilamina. Procainamida.

135. Un paciente varn de 36 aos fue trasladado al servicio de urgencias despus de sufrir un acceso de cefalea aguda, seguido de cada brusca al suelo y acompaado de vmitos. En las semanas previas haba presentado tres episodios similares en los que, tras sentir un dolor intenso en regin frontal, caa desplomado al suelo. En ninguno de estos episodios haba llegado a perder el conocimiento. Segn relataba l mismo, junto con el dolor de cabeza, perda la fuerza en las piernas de forma repentina, recuperndola al cabo de 1-2'. Estos episodios tenan relacin con maniobras de hiperextensin del cuello, o al acostarse en decbito supino. Desde haca unos meses, el enfermo presentaba tambin dolor lumbar sordo de moderada intensidad. En el momento del ingreso el enfermo se encontraba obnubilado, con lenguaje pobre e incoherente. Se detect un trastorno de la motilidad ocular por parlisis de la mirada vertical. En el fondo de ojo haba papiledema bilateral. En cuanto al sistema motor se puso de manifiesto un marcado deficit de fuerza en ambas piernas con hiperreflexia y Babinski + bilateral. No presentaba rigidez de nuca ni otros signos menngeos. En cuanto al dolor lumbar, ste se incrementaba con la presin sobre las apfisis espinosas lumbares. El signo de Lasgue era negativo y no se encontraron signos de radiculopata. De los siguientes, qu diagnstico no emitiras sobre la impresin inicial del cuadro clnico de este paciente? 1. 2. 3. 4. 5. Drop-attack. Sndrome de Parinaud. Hidrocefalia. Hernia discal L4-L5. Sndrome de primera motoneurona.

139. Una es falsa relacionada con el tratamiento del Sndrome de Wernicke-Korsakoff: 1. Debe considerarse como una urgencia mdica y debe iniciarse con 100 mg.de tiamina por va intravenosa o intramuscular durante al menos 5 das. 2. Suele recomendarse la administracin de magnesio en forma sulfato por va intramuscular. 3. Las alteraciones oculomotoras suelen ceder en varias horas, aunque puede persistir cierto grado de nistagmo horizontal o vertical. 4. El tratamiento de la psicosis de Korsakoff consiste en la administracin de tiamina lo antes posible y, aunque la mitad de los pacientes no responden, debe mantenerse el tratamiento para evitar la progresin de las lesiones; la fluvoxamina tambin puede ser til en este sndrome. 5. Est contraindicada la administracin de glucosa por va intravenosa en los pacientes con este sndrome. 140. Enfermo de 63 aos que se ha encontrado

- 14 -

Centro de Estudios Aula Neptuno - GRANADA


www.aulaneptuno.com

Ver.: 10/2001 5. Miotona congnita. 144. Un paciente alcohlico de 48 aos refiere prdida de fuerza muscular, ms acusada en las regiones distales de las extremidades inferiores, parestesias y dolor en las mismas, y sensacin quemante en las plantas de los pies. A la exploracin hay disminucin de sensibilidades superficial y profunda junto a hiporreflexia rotuliana y aqulea. El diagnstico es: 1. 2. 3. 4. 5. Mielinolisis centropontina. Enfermedad de Marchiafava Bignami. Polineuropata alcohlica. Miopata alcohlica. Degeneracin cerebelosa por el alcohol.

bien hasta dos meses antes del ingreso. Comienza con astenia, anorexia, malestar general, tos con escas expectoracin, dolor en punta del costado izquierdo y febrcula. Junto a ello refiere prdida paulatina de fuerza muscular, mialgias generalizadas y dolorimiiento lumbar persistente. Es fumador y diabtico. Exploracin: Mal estado general. Obnubilacin y desorientacin temporo-espacial. Taquicardia y taquipnea. Disminucin generalizada del murmullo vesicular con crepitantes bibasales. Hiperreflexia derecha con Babinski, fuerza muscular disminuida, rigidez de nuca. Fondo de ojo, puncin lumbar y TAC craneal normales. EEG: Ocasionales brotes lentos en hemisferio derecho. Analtica: Glucosa: 207 mg/dl; Calcio: 14 mg/dl; resto parmetros dentro de la normalidad. Gasometra: pH 7,49; PO2: 60 mm.Hg.; PCO2: 27 mm.Hg. Hemocultivos, serologas y sedimento urinario negativos. Rx.de trax: Condensacin basal izquierda, con posible atelectasia parcial. Broncoscopia y biopsia: Metaplasia escamosa con displasia grave. Cul es la explicacin ms probable del cuadro neurolgico? 1. 2. 3. 4. 5. SIADH. Sndrome miasteniforme de Eaton Lambert. Degeneracin cerebelosa subaguda. Encefalopata por hipercalcemia. Encefalopata anxico-isqumica.

145. Un paciente presenta intenso dolor a nivel de D6 que se irradia en cinturn y se incrementa con la tos, estornudos, etc. Adems refiere calambres y debilidad en las piernas.Con la exploracin se demuestra dolor en las apfisis espinosas dorsales, prdida de fuerza global en ambas piernas con franca hiperreflexia, signo de Babinski bilateral y disminucin de la sensibilidad vibratoria. Cul de los siguientes sndromes es el que presenta el paciente? 1. 2. 3. 4. 5. Sind. tabtico. Sind. de compresin radculo medular. Sind. de la arteria espinal anterior. Sind. siringomilico. Sind. cordonal posterior.

141. Un paciente es trado al servicio de Urgencias tras sufrir parada cardiorrespiratoria y posterior reanimacin. Cul de los siguientes hallazgos a la exploracin presupone un peor pronstico de la encefalopata anxica?: 1. 2. 3. 4. 5. Presencia de mioclonias. Ojos ligeramente divergentes. Desarrollo de crisis generalizadas. Patrn respiratorio rtmico. Pupilas arreactivas.

142. Julio B., de 14 aos de edad, acude a tu consulta por debilidad de miembros inferiores de varios aos de evolucin. La exploracin fsica muestra la presencia de pies cavos. Otros familiares presentan un cuadro clnico similar. El EMG revela una velocidad de conduccin motora muy disminuida. La biopsia de nervio demuestra la presencia de una neuropata desmielinizante y formacin en bulbo de cebolla. Pensaras como diagnstico ms probable en: 1. Neuropata amiloidea hereditaria. 2. Neuropata hereditaria sensitivo-motora tipo I. 3. Enfermedad de Fabry. 4. Enfermedad de Refsum. 5. Adrenoleucodistrofia. 143. Las cataratas, calvicie frontal, atrofia testicular, debilidad y consuncin muscular se observan acompaando a: 1. 2. 3. 4. Distrofia miotnica. Distrofia oculofarngea. Distrofia seudohipertrfica. Distrofia facioescapulohumeral.

146. Un varn de 65 aos refiere historia de 3 aos de evolucin caracterizada por debilidad progresiva de miembros inferiores, parestesias de manos y pies y sensacin elctrica desde area cervical irradiada hacia abajo con la flexin del cuello. A la exploracin, amiotrofias en manos, disminucin de la sensibilidad vibratoria e hiperreflexia en miembros inferiores con signo de Babinski. El diagnstico clnico ms probable es: 1. 2. 3. 4. 5. Sndrome de la arteria espinal anterior. Sndrome tabtico. Esclerosis mltiple. Espondilosis cervical. Sndrome de BROWN-SEQUARD.

147. Sobre las conexiones anatmicas de los ganglios basales, indica la afirmacin incorrecta: 1. El cuerpo estriado (caudado-putamen) recibe aferencias corticales, de las neuronas de la sustancia negra y de los ncleos talmicos. 2. El cuerpo estriado emite axones que se proyectan hacia el globo plido y la sustancia negra. 3. Los principales ncleos de salida de los ganglios basales, el segmento interno del globo plido y la porcin reticular de la sustancia negra, emiten proyecciones inhibitorias gabrgicas. 4. Los ganglios basales, al igual que el sistema piramidal, emiten haces que conectan directamente con la mdula espinal. 5. El estriado modula las eferencias de los ganglios basales de dos formas: una directa, me-

- 15 -

Centro de Estudios Aula Neptuno - GRANADA


www.aulaneptuno.com

Ver.: 10/2001 1. 2. 3. 4. 5. Posicin en decbito. Espiracin. Contraccin de msculos esquelticos. Insuficiencia renal. Fibrilacin auricular.

diada por sustancia P y GABA, excitadora sobre los movimientos corticales; y otra indirecta, mediada por encefalina y GABA, inhibidora de los mismos. 148. La primera sinapsis de las vas que vehiculizan la sensibilidad termoalgsica se realiza en: 1. Las interneuronas del asta anterior de la mdula. 2. Los ncleos gracilis y cuneiforme del bulbo. 3. La sustancia gris periacueductal. 4. La lamina marginalis del asta posterior de la mdula. 5. El ncleo ventral posterolateral del tlamo. 149. La mononeuritis en la que resulta imposible flexionar la cadera y extender la rodilla, desaparece o es dbil el reflejo rotuliano y no se pueden subir escaleras ni saltar, corresponde al nervio: 1. 2. 3. 4. 5. Crural. Obturador. Femorocutneo. Citico poplteo interno. Citico poplteo interno.

155. Cul de las siguientes frases define mejor la influencia de la acidosis sobre la excrecin renal de potasio? 1. La acidosis siempre estimula la secrecin renal de potasio. 2. La acidosis siempre inhibe la reabsorcin renal de potasio. 3. La acidosis aguda aumenta la excrecin de potasio, en tanto que la crnica no tiene efecto sobre la misma. 4. La acidosis aguda disminuye la excrecin renal de potasio, en tanto que si persiste varios das se estimula la misma. 5. La acidosis aguda estimula la secrecin renal de potasio, mientras que si dura varios das, la misma queda inhibida. 156. Para una persona que est de pie el centro de gravedad se sita en: 1. 2. 3. 4. 5. Apndice xifoides. Zona periumbilical. Pelvis. 12 vrtebra dorsal. Equidistante entre ambos troquteres.

150. Cul de estos medicamentos se utiliza muchas veces con xito en la profilaxis de la migraa? 1. 2. 3. 4. 5. Vitamina E. Dipiridamol. Tiacidas. Amitriptilina. Clofibrato.

151. La aparicin de una debilidad motora generalizada que se instaura en menos de una hora debe hacer pensar en: 1. 2. 3. 4. 5. Polineuropata inflamatoria aguda. Hipermagnesemia. Polidermatomiositis. Enfermedad de la segunda motoneurona. Distrofia muscular.

157. Si la prevalencia de diabetes en una colectividad de 10.000 habitantes es del 1% y la sensibilidad y especificidad de la glucemia son del 90%, cul es el cmputo aproximado del valor predictivo del test positivo? 1. 2. 3. 4. 5. 6%. 8%. 10%. 50%. 99%.

152. La cantidad de sangre que pasa a travs de un vaso en un tiempo dado y para un determinado gradiente de presin se mide en mL/seg/mm.Hg. Y se denomina: 1. 2. 3. 4. 5. Afluencia. Chorreo. Conductancia. Capacitancia. Impedancia.

158. En la evaluacin de un programa de salud comunitaria, la relacin entre los objetivos alcanzados y los recursos utilizados se denomina: 1. 2. 3. 4. 5. Rendimiento. Eficacia. Rentabilidad. Eficiencia. Efectividad.

159. Qu significa una fraccin etiolgica del 90%? 1. Hay 90 casos que se deben al factor de riesgo. 2. El 90% de los casos de la enfermedad se deben al factor de riesgo. 3. El 90% de los expuestos estn enfermos. 4. El 90% de la poblacin est expuesta al factor de riesgo. 5. El 90% de los expuestos enfermos han enfermado por culpa del F. de riesgo. 160. Cul de estos estudios No puede ser considerado analtico?

153. Como es lgico, pues, la conductancia estar en proporcin inversa a: 1. 2. 3. 4. 5. El gasto cardiaco. La resistencia. La segunda potencia del dimetro. La cuarta potencia del dimetro. La presin en el extremos proximal del vaso.

154. Indica cul de los mecanismos que a continuacin se refieren provoca una reduccin de la precarga del ventrculo izquierdo:

- 16 -

Centro de Estudios Aula Neptuno - GRANADA


www.aulaneptuno.com

Ver.: 10/2001 nal. 3. La diferencia de riesgos absolutos. 4. La diferencia de incidencias a nivel poblacional. 5. El riesgo relativo. 167. Cul de las siguientes relaciones define la capacidad vital forzada? 1. La suma del volumen corriente ms el volumen de reserva espiratorio. 2. La suma del volumen de reserva inspiratorio ms el volumen de reserva espiratorio. 3. La suma del volumen corriente ms la capacidad inspiratoria. 4. La suma de la capacidad inspiratoria ms el volumen de reserva espiratorio. 5. La resta del volumen residual al volumen de reserva espiratorio. 168. Cul de las siguientes es la causa ms frecuente de hipoxemia? 1. Desequilibrio de la relacin ventilacin perfusin. 2. Hipoventilacin. 3. Alteracin de la difusin. 4. Cortocircuito derecha izquierda. 5. Respiracin en grandes alturas. 169. Qu afirmacin de las siguientes es falsa en relacin a las caractersticas diferenciales entre la EPOC tipo enfisema y la EPOC con predominio de bronquitis crnica? 1. La capacidad de difusin suele estar ms disminuida en los pacientes con enfisema. 2. Los episodios de insuficiencia respiratoria son frecuentes en la bronquitis crnica, mientras en el enfisema suceden en fases terminales. 3. La hipertensin pulmonar empeora con el ejercicio en los pacientes con enfisema, y mejora con el mismo en los pacientes con bronquitis crnica. 4. La retraccin elstica es normal en la bronquitis crnica y est disminuida en el enfisema. 5. El cor pulmonale es frecuente en la bronquitis crnica, mientras en los pacientes con enfisema slo se observa en fases muy avanzadas del cuadro. 170. Ante una insuficiencia respiratoria que no responde a la oxigenoterapia al 100%, cul de las siguientes alteraciones patognicas ser la predominante? 1. 2. 3. 4. 5. Desequilibrio V/Q. Efecto shunt. Aumento del espacio muerto anatmico. Embolismo pulmonar. Exudado alveolar.

1. 2. 3. 4. 5.

Transversal. De Cohortes. Prospectivo. Retrospectivo. Casos y controles.

161. Cul de los siguientes componentes del humo del tabaco no se puede considerar por s solo como sustancia carcingena? 1. 2. 3. 4. 5. Hidrocarburos aromticos polinucleares. Indol. Nitrosaminas. Beta naftilamina. Monxido de carbono.

162. Para que una patologa pueda ser escogida como trazador (indicador de calidad) debe tener las siguientes caractersticas, excepto: 1. 2. 3. 4. 5. Estar bien definida. Ser de diagnstico fcil. Ser de baja prevalencia. Tener un impacto funcional definido. Variar con la utilizacin del sistema sanitario.

163. Si afirmamos que "el 80% de los contactos familiares susceptibles de un nio con una enfermedad infecciosa desarrollan esta enfermedad nos estamos refiriendo a: 1. 2. 3. 4. 5. Tasa de prevalencia. Tasa de ataque secundario. Tasa de mortalidad. Tasa de mortalidad ajustada por edad. Tasa de letalidad.

164. El estudio de casos prevalentes en un estudio de casos controles de una enfermedad de elevada letalidad, puede producir: 1. 2. 3. 4. 5. Sesgo de seleccin. Sesgo de informacin. Error aleatorio. Fenmeno de confusin. Sesgo de Berkson.

165. Se aplica un test diagnstico para una enfermedad en dos poblaciones con prevalencias del proceso, respectivamente, del 1% y del 10%. Entonces: 1. El valor predictivo positivo es igual en las dos poblaciones. 2. El valor predictivo positivo es mayor en la primera. 3. EL valor predictivo positivo es mayor en la segunda. 4. La tasa de falsos positivos es mayor en la primera. 5. La tasa de falsos negativos es mayor en la segunda. 166. El 60% de los fumadores con enfermedad coronaria deben la misma al hbito de fumar. Estoy describiendo: 1. La proporcin de riesgo atribuible o fraccin etiolgica en expuestos. 2. La proporcin de riesgo atribuible poblacio-

171. Cul es el signo clnico ms importante a la hora de diagnosticar una obstruccin aguda de la va area superior? 1. Hipoxemia. 2. Cianosis. 3. Estridor.

- 17 -

Centro de Estudios Aula Neptuno - GRANADA


www.aulaneptuno.com

Ver.: 10/2001 grafa de trax: Patrn intersticial bibasal sin adenopatas ni alteraciones pleurales aparentes. Estudio funcional respiratorio: Grave insuficiencia ventilatoria de tipo restrictivo con una CVF del 56%, una CPT del 61% y un Tiffeneau del 85%. Lavado broncoalveolar: Predominio de neutrfilos sin otros datos de inters. En la biopsia pulmonar, cul es el diagnstico ms probable? 1. 2. 3. 4. 5. Fibrosis pulmonar idioptica. Fibrosis qustica. Sarcoidosis. Asbestosis. Lo ms probable es que no se llegue a un dgto.preciso con la biopsia pulmonar.

4. Tiraje intercostal. 5. Hipercapnia. 172. Entre las siguientes colagenosis, la que con ms frecuencia se asocia a enfermedad intersticial pulmonar es: 1. 2. 3. 4. 5. LES. Polimiositis. Esclerodermia. Sjgren primario. Enfermedad de Still.

173. La neumopata intersticial producida por uno de los siguientes frmacos no es dosis dependiente: 1. 2. 3. 4. 5. Amiodarona. Metotrexate. Busulfn. Bleomicina. Nitrofurantona.

176. Qu afirmacin es falsa en relacin con la alveolitis alrgica extrnseca? 1. Una induracin de ms de 10 mm. a los 1015 minutos del contacto con el antgeno es criterio de positividad en las intradermorreacciones. 2. El BAL suele contener un elevado porcentaje de linfocitos T con disminucin del cociente T4/T8. 3. El patrn es inicialmente restrictivo, siendo posteriormente sustituido por un patrn mixto. 4. El tratamiento ms importante pasa por retirar al individuo del contacto con el antgeno responsable. 5. Uno de los datos diferenciales ms importantes con las EPI es la presencia de derrame pleural y adenopatas con mayor frecuencia en las AAE. 177. Uno de los siguientes patrones de calcificacin de un ndulo pulmonar solitario no implica benignidad: 1. 2. 3. 4. 5. Excntrica. Central. Difusa. Laminar. En palomita de maz.

174. Una de las siguientes afirmaciones sobre el tratamiento de la trombosis venosa profunda y el embolismo pulmonar es falsa: 1. La trombosis venosa profunda que permanece limitada a las venas de la pantorrilla deben tratarse con heparinizacin durante 10 das en todos los casos, salvo que el riesgo hemorrgico sea muy alto (enfermos con defecto conocido de la hemostasia). 2. Cuando existe una sospecha fundada de embolia pulmonar basada en los datos clnicos y pruebas usuales de laboratorio debe instaurarse inmediatamente el tratamiento con heparina, sin esperar a la confirmacin diagnstica, a menos que la heparina suponga un grave riesgo para el paciente. 3. Hay que evitar las inyecciones intramusculares de heparina por los hematomas que producen. 4. La falta de trombosis venosa profunda en la flebografa no descarta el diagnstico de embolismo pulmonar. 5. La insercin de un filtro en la vena cava est indicada en pacientes con embolias masivas que no toleraran la recidiva y en pacientes con trombosis venosa demostrada en que est contraindicado el tratamiento anticoagulante. 175. Paciente varn de 35 aos. Fumador de 20-30 cigarrillos da. Empresario de la construccin. Haba tenido contacto espordico con materiales como slice, asbesto y cemento. Desde hace 4 aos presenta disnea de grandes esfuerzos, atribuida inicialmente a su obesidad. La disnea ha ido progresando a lo largo de este tiempo, hasta llegar a ser de mnimos esfuerzos, con episodios de dolor en la regin xifoidea y cianosis labial y ungueal tras el esfuerzo. No haba presentado tos ni expectoracin y tampoco exista historia previa de infecciones respiratorias de repeticin. En la exploracin fsica se objetiv obesidad moderada y evidentes acropaquias en extremidades superiores e inferiores. Auscultacin pulmonar: Hipoventilacin bibasal, con estertores crepitantes inspiratorios. Las determinaciones de laboratorio usuales eran normales. Radio-

178. El sndrome de secrecin inadecuada de ADH, como sndrome paraneoplsico hormonal, se asocia con ms frecuencia a una de las siguientes neoplasias pulmonares: 1. 2. 3. 4. 5. Carcinoma broncognico microctico. Epidermoide. Adenocarcinoma. Carcinoide. Linfoma.

179. La causa ms frecuente de quilotrax es: 1. 2. 3. 4. 5. Linfoma. Traumatismos accidentales o yatrgenos. Linfangiomiomatosis. Artritis reumatoide. Tuberculosis.

180. De las siguientes imgenes radiolgicas, slo una no concuerda con el diagnstico de cncer broncopulmonar:

- 18 -

Centro de Estudios Aula Neptuno - GRANADA


www.aulaneptuno.com

Ver.: 10/2001 4. En el patrn funcional respiratorio destaca un patrn restrictivo con afectacin de la DLCO. 5. Su pronstico es malo debido a la falta de respuesta a la corticoterapia. 185. La falta de una protena microtubular denominada dinena est relacionada con la aparicin de: 1. 2. 3. 4. 5. Traquebroncomegalia. Proteinosis alveolar. Sndrome de Oldfield. Sndrome de Young. Sndrome de los cilios discinticos.

1. 2. 3. 4.

Imagen derrame-atelectasia. Ensanchamiento hiliar. Opacidad velada de un vrtice pulmonar. Ndulo pulmonar solitario con calcificaciones en su interior. 5. Opacidad cavitada con paredes irregulares y muesca.

181. Un paciente consulta por disfagia progresiva para slidos y por dolor interescapular. A la exploracin se observa ptosis palpebral y miosis unilateral. La radiografa de trax comprueba una masa en "reloj de arena" en mediastino posterior. EL diagnstico ms probable es: 1. 2. 3. 4. 5. Quiste broncognico. Persistencia del conducto tirogloso. Tumor neurognico. Sarcoma de Sternberg. Hernia de Bochdaleck.

186. La causa ms frecuente de hemoptisis de intensidad leve-mediana es: 1. 2. 3. 4. 5. La tuberculosis pulmonar. Las ditesis hemorrgicas. Las fstulas arteriovenosas. Las bronquiectasias. Los carcinomas broncognicos.

182. Cul de las siguientes enfermedades pulmonares tiene ms posibilidades de responder al tratamiento glucocorticoideo oral a dosis moderadas? 1. 2. 3. 4. 5. Fibrosis pulmonar idioptica. Proteinosis alveolar pulmonar. Linfangioleiomiomatosis. Amiloidosis. Neumona eosinoflica crnica.

187. Con respecto a la resistencia a la insulina por alteracin de los receptores tipo A es falso: 1. 2. 3. 4. 5. Se asocia a acantosis nigricans. Es ms frecuente en mujeres jvenes. Con frecuencia presentan ovario poliqustico. Se debe a Ac.contra el receptor de insulina. El nmero de receptores de insulina est disminuido.

183. Cul es falsa sobre el enfisema bulloso? 1. Es un conjunto de alteraciones parenquimatosas que cursan con la formacin de uno ms espacios areos intraparenquimatosos con un dimetro superior a 1 centmetro. Debe distinquirse de las vesculas pleurales (dentro de la pleura visceral) y de los quistes (revestidos de epitelio). 2. Distinguimos las bullas tipo I, subpleurales, no asociadas con enfisema y de cuello estrecho; las tipo II, con cuello ancho, localizacin en lbulo medio y asociacin con enfisema panacinar; y las tipo III, casi iguales que las anteriores pero con base mucho ms ancha y localizacin en cualquier zona del pulmn. 3. Las bullas se ven mejor en las radiografas efectuadas en espiracin. La gammagrafa valorar la compresin sobre parnquima adyacente. 4. Las complicaciones son infeccin, hemorragia y neumotrax. 5. El nombre de enfisema "bulloso" se debe a la rapidez (bulla) con que el paciente acude a consulta al notar los primeros sntomas. 184. Respecto a la bronquiolitis obliterante con neumona organizada, cul de las siguientes afirmaciones es falsa? 1. Se caracteriza por la presencia en la luz de los bronquiolos respiratorios, conductos alveolares y alveolos, de un tejido fibroblstico de matriz mixoide con clulas inflamatorias. 2. Su forma de presentacin es tpicamente seudogripal y subaguda. 3. En la radiografa de trax se observan infiltrados alveolares perifricos cambiantes. 188. Los mecanismos por los que el stress fsico o emocional desencadenan cetoacidosis al aumentar la adrenalina y la noradrenalina son los citados a continuacion, salvo uno. Selalo: 1. Estimular el glucagon. 2. Bloquear la liberacin de la escasa insulina residual. 3. Inhibir el transporte de glucosa a los tejidos perifricos. 4. Estimulacin de la gluconeognesis. 5. Inhibicin de la sntesis de glucocorticoides por la corteza suprarrenal. 189. Ante un individuo con DMNID, de larga evolucin y mal control metablico, que presenta glucemia de 259 mg%, cetoamina (Hb A1c) de 12% y aclaramiento de creatinina de 60 ml/min, el tratamiento menos apropiado sera: 1. 2. 3. 4. 5. Tolbutamida y acarbosa. Acetohexamida y dieta estricta. Biguanida y acarbosa. Clorpropamida y acarbosa. Insulina NPH.

190. Un hombre de 32 aos con diabetes insulinodependiente ingresa en el hospital para someerse a un estudio por marcada insuficiencia renal. Tiene una urea de 123 mg/dl y una creatinina de 9.8 mg/dl. Dos aos antes de su ingreso, se observ que tena proteinuria sin insuficiencia renal. Meses antes del ingreso tena una urea de 30 mg/dl y una creatinina de 1,8 mg/dl. Cul de las siguientes es la causa me-

- 19 -

Centro de Estudios Aula Neptuno - GRANADA


www.aulaneptuno.com

Ver.: 10/2001 2. 3. 4. 5. Probenecid y biicarbonato. Alopurinol. Colchicina. Sulfinpirazona.

nos probable de la insuficiencia renal de este paciente? 1. 2. 3. 4. 5. Atona vesical. Infeccin urinaria. Hipertensin incontrolada. Glomerulonefritis rpidamente progresiva. Evolucin natural de la nefropata diabtica.

196. Ante una hipercalcemia grave (Ca mayor a 18 mgr/dl), que requiera un rpido control del calcio, seale qu frmaco es el de primera eleccin: 1. 2. 3. 4. 5. EDTA. Calcitonina. Mitramicina. Prednisona. Fosfatos.

191. Paciente de 34 a. diabetica insulin dependiente desde hace 17 a. ingresa en hospital con cuadro de coma. La exploracion fisica muestra edema periorbitario, secrecion nasal sanguinolenta y signos de necrosis de mucosa nasal. La bioquimica y estudio de gases arteriales suieren cuadro de cetoacidosis. Cual de los siguientes antibioticos seria de primera eleccion: 1. 2. 3. 4. 5. Ceftrixona. Piperacilina-tazobactam. Anfotericina B. Ketoconazol. Rifampicina.

197. Slo una de estas condiciones no se cumple en el basalioma: 1. 2. 3. 4. 5. No afecta mucosas. No tiene lesiones preneoplsicas. No produce metstasis. No se acompaa de adenopatas. Es el tumor maligno ms frecuente de la piel.

192. Cul de las siguientes manifestaciones es menos tpica de hipercolesterolemia? 1. 2. 3. 4. 5. Arco corneal. Xantomas eruptivos. Xantomas planos. Xantomas tendinosos. Xantelasmas.

198. Un hombre de 57 aos ha notado la aparicin de hiperpigmentacin difusa en su piel. Todas las siguientes explicaciones son posibles, excepto: 1. 2. 3. 4. 5. Una reaccin farmacolgica. Un pnfigo vulgar. Una cirrosis biliar. Una tumoracin maligna subyacente. Un melanoma metastsico.

193. Seale cul de las siguientes formas clnicas de porfiria se transmite con caracter autosmico recesivo: 1. 2. 3. 4. 5. Porfiria cutnea tarda. Porfiria aguda de DOSS. Porfiria variegata. Porfiria aguda intermitente. Todas las anteriores.

199. Qu afirmacin es falsa en el Sndrome de Sweet? 1. La mejor alternativa al tratamiento cortociodeo es el yoduro potsico. 2. Ante un paciente con Sndrome de Sweet es necesario considerar la posible asociacin con hemopatas malignas. 3. La presencia de trombosis y necrosis fibrinoide hace que se incluya dentro del grupo de las vasculitis leucocitoclsicas. 4. Es tpico el infiltrado drmico por polinucleares neutrfilos. 5. Se han descrito en l ANCA con patrn perinuclear. 200. La alopecia areata es una forma de alopecia no cicatrizal que se asocia a ciertas enfermedades. Indica la falsa: 1. 2. 3. 4. 5. Sndrome de Down. Enfermedad de Addison. Inmunodeficiencias congnitas. Anemia perniciosa. Cncer de mama metastatizado.

194. Cul es el perfil lipmico de los pacientes afectos de disbetalipoproteinemia familiar? 1. Elevacin del colesterol y de los triglicridos en proporcin 2:1 a favor del colesterol. 2. Elevacin de los triglicridos con cifras normales de colesterol. 3. Elevacin del colesterol y de los triglicridos a cifras similares (relacin 1:1). 4. Elevacin del colesterol con cifras normales de triglicridos. 5. Elevacin de los triglicridos y del colesterol en proporcin 3:1 a favor de los triglicridos. 195. Un varn de 67 aos de edad con artritis crnica ha expulsado un clculo de cido rico despus de un episodio de clico renal. En el estudio se observa que hay mltiples clculos radiolcidos en la pelvis renal izqda., la excrecin de cido rico es de 90 mg/da, la concentracin srica de rico es de 9,8 mg/dL, concentracin srica de creatinina 1,8 mg/dL, y cristales de urato monosdico en la rodilla izqda. El frmaco ms adecuado para tratamiento a largo plazo de este paciente es: 1. Probenecid.

201. La lesin histopatolgica ms caracterstica del eczema es: 1. 2. 3. 4. 5. Paraqueratosis. Hiperepidermopoyesis. Disgranulosis. Acantolisis. Espongiosis.

- 20 -

Centro de Estudios Aula Neptuno - GRANADA


www.aulaneptuno.com

Ver.: 10/2001 la lcera gstrica. 3. El tipo histolgico intestinal es el ms frecuente, tiene localizacin antral, precncer asociado y diferencias geogrficas. 4. Debe realizarse diagnstico diferencial con la linitis plstica, proceso benigno que da imgenes radiolgicas similares al adenocarcinoma infiltrante. 5. La aparicin del signo de Leser Trelat no es sinnimo de diseminacin metastsica. 207. Cul de las siguientes pruebas tiene mayor sensibilidad y especificidad para diagnosticar causas mucosas de malabsorcin? 1. 2. 3. 4. 5. Cuantificacin de grasa fecal. Prueba de triolena marcada con C-14. Prueba de D-xilosa. Radiografa de intestino delgado. Biopsia de intestino delgado.

202. Respecto a los divertculos esofgicos, una no es cierta: 1. El tratamiento del divertculo de Zenker, cuando es grande, consiste en diverticulectoma y miotoma del cricofarngeo. 2. Los medioesofgicos suelen producirse por retracciones o por alteraciones motoras de esfago. 3. Los epifrnicos asientan inmediatamente por encima de la unin esofago-gstrica. 4. Los medio-esofgicos requieren tratamiento quirrgico precoz, por el rieesgo de perforacin de esfago. 5. La neumonitis por aspiracin es la complicacin ms frecuente del de Zenker. 203. No se considera indicacin de endoscopia en pacientes con reflujo esofagogstrico: 1. Reflujo asociado a hipersecrecin gstrica. 2. Reflujo con disfagia, anemia o prdida de peso. 3. Los sntomas persisten a pesar del tratamiento. 4. Pacientes de ms de 40 aos con pirosis de ms de 10 aos de evolucin. 5. Si se valora la posibilidad de una intervencin quirrgica antirreflujo. 204. En cul de estos pacientes se puede realizar estudio con endoscopia digestiva alta (EDA)? 1. Paciente en estado de shock por sangrado masivo. 2. Paciente con antecedentes de cardiopata isqumica y IAM hace dos aos. 3. Paciente con enfisema en fase terminal e insuficiencia respiratoria severa. 4. Paciente con sospecha de diseccin de aorta. 5. Paciente con dolor epigstrico brusco y neumoperitoneo en la Rx.simple de abdomen. 205. Hace cuatro meses, en un varn de 36 aos de edad con lcera pptica, se efectu una anastomosis Billroth II. Hoy vuelve para valoracin de una lcera en la boca anastomtica. La concentracin srica de gastrina en el ayuno es de 350 pg/mL; cinco minutos despus de la administracin intravenosa de secretina, la concentracin srica de gastrina es de 100 pg/mL. Es necesario hacer saber al paciente que el tratamiento ms adecuado para su padecimiento es: 1. Vagotoma total. 2. Gastrectoma total. 3. Reseccin del antro distal fijado al mun duodenal. 4. Laparotoma para buscar una neoplasia productora de gastrina. 5. Tratamiento mdico con anticidos lquidos 206. Qu afirmacin es falsa sobre el cncer gstrico? 1. El cncer gstrico es el que afecta a las capas mucosa y submucosa, independientemente de la afectacin ganglionar. 2. Aunque la lcera gstrica no sea considerada una lesin preneoplsica, s lo es la ciruga de

208. En un paciente con sospecha de cncer colorrectal se realiza rectosigmoidoscopia, cuyo resultado es negativo. En general, y considerando fiabilidad, complicaciones y eficiencia, cul es la siguiente prueba que se debe practicar? 1. Colonoscopia. 2. Puede optarse por colonoscopia o por el enema opaco con similar justificacin. 3. Determinacin de CEA. 4. Enema opaco. 5. Laparotoma exploradora. 209. Las enfermedades hepticas agudas o crnicas pueden producir tambin malabsorcin de ciertos nutrientes. Con respecto a ello, es falso: 1. La prueba de la absorcin de la D-xilosa muestra un resultado patolgico. 2. Las biopsias de mucosa del intestino delgado no aportan ningn dato significativo. 3. La esteatorrea se debe a una alteracin de la sntesis heptica o de la excrecin de sales biliares conjugadas. 4. Pueden producirse alteraciones de la absorcin de la vitamina D y el calcio, especialmente frecuente en los casos de cirrosis biliar primaria. 5. Los pacientes con hepatopata de origen alcohlico tambin pueden tener alteracin de la funcin exocrina del pncreas. 210. Paciente de 74 aos que se despierta por la noche con intenso dolor en fosa iliaca izquierda, de tipo constante y sin irradiaciones, acompaado de fiebre. A la exploracin hay signos evidentes de peritonismo, con importante distensin abdominal. Indicars todos los siguientes pasos diagnsticos y teraputicos, excepto: 1. 2. 3. 4. 5. Enema opaco. Rx.simple de abdomen. Hemograma y frmula leucocitaria. Dieta absoluta. Aspiracin naso-gstrica.

211. En el caso clnico del anterior paciente, todos estos datos, excepto uno, constituirn indica-

- 21 -

Centro de Estudios Aula Neptuno - GRANADA


www.aulaneptuno.com

Ver.: 10/2001 con varices y grado C de Child. 217. Cul de los siguientes datos analticos es ms especfico del origen alcohlico de una hepatitis aguda? 1. Disminucin de los niveles de gammaglutamil transpeptidasa. 2. Elevacin de la bilirrubina srica. 3. Hipertransaminasemia superior a 500 U/L. 4. Glucemia superior a 300 mg/dl. 5. Cociente AST/ALT superior a 2. 218. Un hombre de 65 aos con una cirrosis postnecrtica antigua, estable y comprobada por biopsia presenta dolor abdominal en el cuadrante superior derecho e hinchazn abdominal. No tiene fiebre. En la exploracin fsica se advierten eritema palmar, telangiectasias estrelladas e ictericia ligera. El abdomen est distendido, con matidez cambiante y son palpables un borde heptico firme y doloroso a tres traveses de dedo del reborde costal y el polo inferior del bazo. Sobre el hgado se ausculta un ruido dbil. En el laboratorio hay: Hto: 34%, recuento leucocitario: 4300/mm3; plaquetas: 104.000 /mm3; albmina srica: 2,6 g/dl; fosfatasa alcalina: 400 U/L. En la paracentesis se extrae un lquido teido de sangre. El marcador srico ms especficamente asociado al proceso de este hombre: 1. 2. 3. 4. 5. Anticuerpos antinucleares. Alfa-fetoprotena. Anticuerpos antimitocondriales. 5-nucleotidasa. Gonadotropina corinica.

cin de ciruga. Seala el error: 1. 2. 3. 4. 5. Niveles hidroareos en la Rx.simple. Absceso local. Fistulizacin. Peritonitis. Oclusin.

212. Y dicha intervencin se realizar mediante: 1. 2. 3. 4. Proctocolectoma total. Drenaje externo de la lesin. Hemicolectoma izquierda. Reseccin del segmento y anastomosis termino-terminal. 5. Reseccin del segmento ms colostoma y anastomosis en un segundo tiempo.

213. Todos los parmetros siguientes EXCEPTO uno, tiene valor pronstico en la pancreatitis aguda. Indique cul: 1. 2. 3. 4. 5. Edad. Cifra de amilasemia. Hipocalcemia. Acidosis. Leucocitosis.

214. El tratamiento ms comunmente propuesto para los pacientes que sufren pancreatitis aguda recidivante como consecuencia de un pncreas divisum es: 1. Pancreatectoma ceflica. 2. Tcnicas de derivacin quirrgica como la coledocoduodenostoma. 3. Pancreatectoma dorsal. 4. Papilotoma de la papila accesoria (drenaje del conducto de Santorini). 5. Trasplante pancretico. 215. Todas las afirmaciones siguientes son ciertas respecto a la cirrosis biliar primaria, excepto una: 1. La prueba de los anticuerpos antimitocondriales es positiva en ms del 90% de los pacientes. 2. Es frecuente una cifra elevada de crioprotena srica. 3. La mayor parte de los pacientes son mujeres. 4. La D-penicilamina es un tratamiento muy eficaz. 5. La artritis reumatoide, el sndrome CREST y la esclerodermia aparecen ms frecuentemente en los pacientes con CBP. 216. Cul de las siguientes no es una indicacin (aceptada o posible) de la derivacin portosistmica percutnea intraheptica (DPPI)? 1. Recidiva hemorrgica por varices esofagogstricas en paciente con anastomosis esplenorrenal malfuncionante. 2. Segunda recidiva significativa en un paciente tratado mediante esclerosis selectiva de varices. 3. Ascitis refractaria al tratamiento diurtico. 4. Episodio hemorrgico que no cesa con drogas vasoactivas y profilaxis endoscpica. 5. Profilaxis primaria del sangrado en pacientes

219. Paciente de 55 aos, obesa, diabtica y HTA que consulta por dolor en hipocondrio derecho junto con anorexia, nuseas y vmitos y fiebre de 37,8 C. A la exploracin fsica el hipocondrio derecho es doloroso, sobre todo a la palpacin en inspiracin profunda. Analtica: 12.000 leucocitos con desviacin izquierda; GOT 132 U/L; GPT 156 U/L; BRR total 2,7 mg/dl; F. alcalina 300 U/L; amilasa 187 U/L. La Rx de abdomen es inespecfica. Diagnstico (primera sospecha): 1. 2. 3. 4. 5. Pancreatitis aguda. Colangitis. Carcinoma de vescula biliar. Hepatitis aguda. Colecistitis aguda.

220. A propsito del colangiocarcinoma es falso: 1. La localizacin ms frecuente es en coldoco distal. 2. La edad mediade afectacin son los 60 aos. 3. Afecta por igual a ambos sexos. 4. No presenta facilidad para las metstasis. 5. El sntoma principal es la ictericia. 221. Todo lo siguiente salvo una entidad se relaciona con la colangitis esclerosante: 1. EII. 2. Fibrosis mediastnica. 3. Fibrosis pulmonar.

- 22 -

Centro de Estudios Aula Neptuno - GRANADA


www.aulaneptuno.com

Ver.: 10/2001 geno se suelen caracterizar por: 1. No depender de variaciones diurnas y estacionales. 2. Pronstico desfavorable con teraputicas biolgicas. 3. Agravamiento de su sintomatologa al caer el da. 4. Desencadenante previo. 5. Inhibicin psicomotriz. 227. Slo una de estas afirmaciones no concuerda con las caractersticas de la alucinosis alcohlica: 1. Suele consistir en percepciones auditivas sin objeto real. 2. Sucede en el espacio exterior al paciente. 3. La temtica de las voces suele estar relacionada con ideas de grandeza autorreferenciales. 4. Son correctamente criticadas por el paciente. 5. No forman parte del delirium tremens. 228. La bulimia nerviosa se caracteriza por todo lo siguiente, excepto: 1. Episodios recurrentes de ingesta masiva de alimentos. 2. Falta de conciencia de anormalidad de la conducta. 3. Finalizacin de la ingesta con dolor abdominal, hipersomnia o vmito. 4. Normopeso o ligero sobrepeso, rara vez obesidad. 5. El prototipo es una mujer de 14 a 40 aos. 229. Qu afirmacin es falsa sobre el autismo? 1. La enfermedad es 3 veces ms frecuente en varones que en mujeres. 2. Hay deterioro de las relaciones afectivas y sociales. Los nios rehyen el contacto fsico. 3. Es frecuente la inversin pronominal, hablando de ellos mismos como "t" o "l". 4. Es rara la aparicin de retraso mental en la evolucin de los nios autistas. 5. El trastorno suele comenzar antes de los 30 meses de edad. 230. Hemos seleccionado una frase para definir cada uno de los siguientes trastornos de la personalidad. Cul de ellas est relacionada inadecuadamente? 1. No se dedican a profesiones que supongan un contacto interpersonal significativo, por miedo a quedar en ridculo delante de los dems: Trastorno por evitacin. 2. La toma de decisiones se postpone por dificultad en tomarlas; tienen una devocin excesiva al trabajo y la productividad, rechazando las diversiones y las amistades: El trastorno es el pasivo-agresivo. 3. Tienden a ser irritables y agresivos, implicndose en peleas fsicas incluso con la familia. Son frecuentes las conductas temerarias sin preocupacin por la seguridad personal: Trastorno antisocial. 4. Tienden a la explotacin interpersonal, valindose de los dems para sus propios fines,

4. Fibrosis retroperitoneal. 5. Seudotumores orbitarios. 222. En un enfermo con Enfermedad por Reflujo en quien se demuestra la existencia de un esfago de Barrett, la pauta ms aceptada de entre las que se mencionan es: 1. Tratamiento quirrgico del reflujo con fundusplicatura y reseccin segmentaria del segmento esofgico que incluye la zona de metaplasia. 2. Tratamiento mdico con ranitidina, 150 mg. al acostarse (por la noche, se entiende) y revisiones con radiologa baritada cada ao. 3. Tratamiento de erradicacin de Helicobacter pylori sin necesidad de seguimiento posterior. 4. Esofaguectoma distal con esofagogastroplastia. 5. Tratamiento mdico con omeprazol 40 mg./da y seguimiento con endoscopia y biopsia cada 2 aos. 223. Un paciente diagnosticado de poliarteritis nodosa presenta un cuadro doloroso abdominal con cierre intestinal, vmitos y exacerbacin de ruidos. EN la radiografa se observan asas intestinales en "pilas de monedas". El diagnstico es: 1. 2. 3. 4. 5. Infarto mesentrico. Colitis isqumica segmentaria. Hemorragia intramural. Invaginacin intestinal. Linfoma intestinal.

224. Paciente de 49 aos, que consulta por presentar tristeza vital sin que, segn cuenta, tenga ninguna razn para ello. Su personalidad previa era normal y su madre tambin present un cuadro afectivo. Dice encontrarse peor por las maanas, despertndose todos los das alrededor de las 5 A.M. En las pruebas complementarias se observa disminucin de la latencia REM, ausencia de supresin con dexametasona y disminucin del MHPG urinario. Cul es cierta? 1. La historia familiar indica que la paciente presenta una depresin neurtica. 2. Los potenciales evocados de esta paciente sern, con toda probabilidad, normales. 3. Tras el tratamiento farmacolgico no se producir hipomana yatrgena, al tratarse de una distimia. 4. Hay muchas probabilidades de que esta paciente presente una psicosis maniaco depresiva. 5. La ausencia de ideas delirantes al inicio del cuadro descarta la depresin mayor. 225. Entre los efectos secundarios del tto.con carbonato de litio no figura: 1. 2. 3. 4. 5. Hipotiroidismo. Bloqueo de la accin de la ADH. Temblor. Prdida de peso. Sntomas gastrointestinales.

226. Las depresiones con mayor contenido end-

- 23 -

Centro de Estudios Aula Neptuno - GRANADA


www.aulaneptuno.com

Ver.: 10/2001 3. Suele diseminarse por va ganglionar. 4. Es el cncer tiroideo ms frecuente. 5. Puede degenerar hacia el cncer anaplsico. 237. Qu afirmacin es cierta sobre el cncer medular tiroideo? 1. El CEA es el marcador bioqumico de las formas peor diferenciadas. 2. Es una de las manifestaciones ms frecuentes de la MEN tipo I. 3. El screening a familiares se basa en la PAAF. 4. Ultimamente su tratamiento se basa en la hemitiroidectoma. 5. Su manifestacin ms frecuente es la tetania por hipocalcemia. 238. De las siguientes, cul es la forma de expresin clnica ms frecuente del hiperparatiroidismo? 1. 2. 3. 4. 5. Asintomtica. Nefropata intersticial. Osteopenia difusa. Anorexia. HTA.

debido al sentimiento grandioso de su propia importancia, exagerando sus logros y capacidades: Trastorno narcisista. 5. Un progenitor del sexo contrario fuerte y dominante y uno del mismo sexo dbil, sumiso y pasivo predisponen al trastorno paranoide de la personalidad. 231. Uno de los siguientes ansiolticos est exento del riesgo de dependencia: 1. 2. 3. 4. 5. Buspirona. Loracepam. Alprazolam. Clobazam. Oxacepam.

232. Los inhibidores de la MAO estn indicados en todos estos trastornos, excepto: 1. 2. 3. 4. 5. Crisis de angustia. Depresiones atpicas. Dolor crnico. Depresiones endgenas (segunda intencin). Profilaxis de los trastornos bipolares.

233. La presencia de episodios de hiperactividad sexual, con exaltacin de nimo e insomnio, junto a temporadas de hipersomnia, desapetencia y pensamientos nihilistas, es tpica de: 1. 2. 3. 4. 5. Ciclotimia. Parasomnia. Astenia primaveral. Trastorno esquizoafectivo. Trastorno disociativo.

239. Joven de 19 aos, embarazada de 4 meses, que presenta amenorrea (esto parece bastante lgico), astenia, apata, involucin mamaria, prdida de vello y de pigmentacin cutneos. Entre las determinaciones hormonales destacan: hGH: 0,1 ng/mL; PRL: 3 ng/mL; ACTH: 7 pg/mL; FSH: 1,1 mU/mL. La causa ms probable de su trastorno es: 1. 2. 3. 4. 5. Sarcoidosis. Craneofaringioma. Adenoma productor de prolactina. Sndrome de Sheehan. Hipofisitis linfoctica.

234. En cul de los siguientes pacientes sera ms peligrosa una intoxicacin aguda por Acetaminofen ? : 1. 2. 3. 4. 5. IRC en programa de Hemodilisis. AR en tto con Penicilamina. Dficit de G-6-P-DH. Portador sano de VHB. TBC en tto con I+R+ETH.

240. Un varn de 13 aos que consulta por talla baja presenta: una edad sea de 11 aos, talla en el percentil 3, velocidad de crecimiento de 4.5 cm. en el ltimo ao y desarrollo sexual G1, P1-2 (Tanner). Su impresin inicial sera: 1. 2. 3. 4. 5. Hipogonadismo hipogonadotrpico. Dficit de GH. Hipotiroidismo. Disfuncin neurosecretora. Retraso constitucional de crecimiento y desarrollo.

235. Un varn de 27 a. con antecedentes de alcoholismo, esquizofrenia y en tto. crnico con Fenotiacinas comienza con clnica de hipersudoracin, hipersialorrea, taquicardia, temperatura de 41,5 oC, polipnea, signos de colapso cardiovascular y de insuf. renal aguda. En estos momentos todo lo siguiente estara indicado EXCEPTO : 1. Suspender la medicacin neurolptica. 2. Desnudar al paciente y colocar paos de agua fra. 3. Instaurar tto. iv. con Dantroleno. 4. Administrar Bromocriptina oral. 5. Colocar va endovenosa y pautar Procainamida : 0,5 mgr/Kgr. 236. No es una caracterstica del cncer papilar tiroideo: 1. Suele presentarse en zonas de bocio endmico. 2. A favor de su dgto. est la presencia de calcificaciones.

241. La prueba funcional ms usada en el dgto.de la enf.de Addison es la de: 1. 2. 3. 4. 5. Supresin con dexametasona. Metopirona. Estmulo con CRH. Estmulo con ACTH. Aminoglutetimida.

242. Cul de estas formas clnicas de S.de Cushing no es ACTH dependiente? 1. 2. 3. 4. Por tratamiento corticoideo. Por el alcohol. Enfermedad de Cushing. Secundario a cncer broncopulmonar.

- 24 -

Centro de Estudios Aula Neptuno - GRANADA


www.aulaneptuno.com

Ver.: 10/2001 5. Alfa metil dopa. 249. La presencia en un nio de aftas orales, lesiones cutneas de bordes policiclicos, hipotensin, crisis epilpticas tipo Gran Mal, astenia, anorexia e hiperpigmentacin podra corresponderse con: 1. 2. 3. 4. 5. S. de DIGEORGE. Esclerosis tuberosa. STURGE-WEBER. S. pluriglandular inmune. Epilepsia rolndica.

5. Hipotalmico. 243. Uno de los siguientes pasos en la sntesis de los esteroides suprarrenales est catalizado por la enzima 11 beta-hidroxilasa: 1. 2. 3. 4. 5. 11-desoxicortisol - Cortisol. Progesterona - 11- desoxicorticosterona. 17-hidroxiprogesterona - 11- desoxicortisol. Pregnenolona - Progesterona. 18-hidroxicorticoesterona - Aldosterona.

244. Paciente en estudio por debilidad y mareos en el que se constata sodio de 137 mEq/L, potasio de 4,4 mEq/L, cortisol de 6 mg/dL, ACTH de 7 pg/mL. El diagnstico ms probable es: 1. 2. 3. 4. 5. Cushing yatrognico. Enf.de Addison. Insuficiencia corticosuprarrenal secundaria. ACTH ectpica. Los resultados son los de una persona sana.

250. Si una paciente presenta infantilismo sexual, talla baja y cintillas gonadales, podemos diagnosticarle de: 1. 2. 3. 4. 5. S. de TURNER. S. de NOONAN. S. de BONNEVIE. Disgenesia gonadal. S. del ovario resistente.

245. Paciente de 25 a. Obesa,con abundante pelo de distribucion masculina, amenorrea; en la exploracion ambos ovarios palpables amentados de tamano. Androgenos aumentados y relacion lh/fsh > 3 sugiere diagnostico: 1. 2. 3. 4. 5. Sind. Kallman. Sind. Stein-Levental. Sind. Turner. Sind. Bourneville-Urich. Sind. Noonan.

251. El adelgazamiento del Cuerpo Mucoso de Malpighio suprapapilar asociado a hiperacantosis en las crestas interpapilares (interpapili, para los amigos) es caracterstico de: 1. 2. 3. 4. 5. Epidermolisis ampollosa. Dermatitis herpetiforme. Liquen plano. Psoriasis. Sndrome de Sweet.

246. La presencia de hiperpigmentacin en un tumor pancretico productor de hormonas debe hacernos sospechar en: 1. 2. 3. 4. 5. Insulinoma. Glucagonoma. Gastrinoma. Vipoma. Somatostatinoma.

252. Respecto a los cidos grasos, cul de las siguientes afirmaciones es correcta? : 1. Su metabolismo celular es dependiente de vitamina B5 o Ac. Pantotnico. 2. Para un mismo nmero de tomos de C, el punto de fusin de los saturados es inferior que los poliinsaturados. 3. Se absorben mediante transporte activo a nivel del duodeno. 4. La molcula iniciadora o cebadora es el Malonil-Co A. 5. El Ac. Oleico y el Palmtico son ac. esenciales. 253. Cul de las siguientes propiedades no es caracterstica de la fraccin C5a del complemento? : 1. Es una anafilotoxina. 2. Induce la secreccin de IL-2 por los macrfagos. 3. Contrae la fibra muscular lisa. 4. Aumenta la permeabilidad vascular. 5. Degranula los basfilos con la consiguiente secreccin de histamina y sustancias vasoactivas. 254. No se considera dentro de las indicaciones de la coronariografa: 1. Pacientes con angina crnica estable o inestable, con enorme sintomatologa a pesar del tratamiento mdico, en los que se considera la revascularizacin. 2. Pacientes en los que se sospechan lesiones

247. Despus de realizar un tratamiento con medio miligramo de dexametasona cada seis horas durante dos das, la cortisolemia es de 3,2 mcg/dl. Cmo podemos traducir este hallazgo? 1. Con toda seguridad, el paciente no presenta una hiperfuncin glucocorticoidea. 2. El paciente es diagnosticado de Sndrome de Cushing. 3. El cuadro es de un hipercortisolismo y, con toda probabilidad, producido por un adenoma hipofisario. 4. Si la ACTH est elevada, casi seguro nos hallamos ante una produccin ectpica de ACTH. 5. El origen suprarrenal es el ms probable. 248. Cul de los siguientes tratamientos no produce como efecto secundario hiperprolactinemia? 1. 2. 3. 4. Verapamil. Antidepresivos tricclicos. Combinacin de levodopa y carbidopa. Butirofenonas.

- 25 -

Centro de Estudios Aula Neptuno - GRANADA


www.aulaneptuno.com

Ver.: 10/2001 dente; asmismo, la afectacin de las ramas principales de la aorta es ms frecuente en su origen que a nivel distal. 257. El principal factor pronstico en el sndrome de aplastamiento es: 1. El lugar en que se est produciendo la compresin. 2. La funcin cardiorrespiratoria del accidentado. 3. El tiempo de isquemia renal. 4. La produccin de gangrena de miembros. 5. La aparicin de dficits neurolgicos focales. 258. La aparicin de cuadriparesia espstica en un recin nacido tiene lugar con MAYOR frecuencia como complicacin de: 1. 2. 3. 4. 5. Prematuridad. Asfixia neonatal. Kernicterus. Meningitis neonatal. Deshidratacin.

del tronco principal izquierdo o de tres vasos coronarios por la presencia de signos de isquemia grave en las pruebas no invasoras, sin tener en cuenta la existencia o gravedad de los sntomas. 3. Pacientes con clnica sugerente de angina en quienes no se puede realizar ergometra por estar limitados fsicamente por enfermedad vascular perifrica o enfermedad musculoesqueltica. 4. Pacientes de alto riesgo despus de un infarto de miocardio debido a la recidiva de la angina, insuficiencia cardiaca, contracciones ventriculares prematuras frecuentes o signos de isquemia en la prueba de esfuerzo. 5. Pacientes con molestias precordiales claramente sugerentes de angina, con prueba de esfuerzo con talio negativa y en las que se requiere un diagnstico definitivo. 255. Con respecto a la absorcin, transporte y metabolismo del hierro, hay una falsa. Selala: 1. La absorcin del hierro tiene lugar preferentemente en el duodeno y la porcin proximal del yeyuno, y est muy favorecida por la acidez gstrica. 2. La absorcin de hierro inorgnico est muy influenciada por los productos de la dieta que pueden quelar dicho elemento. As, el citrato y el ascorbato forman con l complejos solubles que facilitan su absorcin; sin embargo, los taninos del t y los fosfatos dificultan la misma al formar complejos muy ligados con el mismo. 3. El hierro recin absorbido se acopla (con perdn) a la transferrina srica, la cual puede fijar dos tomos de hierro. 4. El exceso de hierro puede acumularse en el organismo en forma de ferritina (con hierro frrico), de la cual entra y sale de manera metablicamente controlada. 5. Cuando es necesario utilizar el hierro de los depsitos, ste pasa a sangre formando una segunda protena transportadora llamada hemosiderina. 256. Una de las siguientes caractersticas anatomo patolgicas de las vasculitis es incorrecta. Indcala: 1. La lesin en la PAN es una inflamacin necrotizante de las arterias de pequeo y mediano calibre; cuando se observa afectacin de las vnulas sugiere un sndrome de superposicin. 2. Las principales diferencias de la PAN con el Churg Strauss radican en que sta ltima afecta al pulmn, a vnulas, muestra granulomas intra y extravasculares e infiltracin tisular por eosinfilos. 3. Los vasos que con ms frecuencia se afectan en las vasculitis por hipersensibilidad son las vnulas postcapilares; se observa en ellas cariorrexis en los neutrfilos y todas las lesiones estn en idntica fase evolutiva. 4. En la biopsia renal de pacientes con granulomatosis de Wegener y glomerulonefritis no es frecuente observar granulomas. 5. La arteria que se afecta ms frecuentemente en la Arteritis de Takayasu es la aorta descen-

259. Enfermo que sufre una ceguera repentina de un ojo, sin dolor ni signos externos, cul es la causa ms probable? 1. 2. 3. 4. 5. Oclusin de la arteria central de la retina. Degeneracin macular. Glaucoma. Iritis. Retinoblastoma.

260. Cul de estos pacientes te despierta menos sospecha de padecer una infeccin por citomegalovirus? 1. Paciente ADVP, VIH (+), que presenta coriorretinitis. 2. Paciente trasplantado renal con dolor torcico y disnea. 3. Lactante con sndrome mononuclesico. 4. Joven de 16 aos con fiebre, exantema, adenopatas, esplenomegalia y mononucleosis. 5. Paciente VIH (+) con astenia, hipotensin, hiponatremia e hiperpotasemia.

- 26 -

CORRECCION n de Preg. CONTESTADAS: n de Preg. ACERTADAS: n de Preg. FALLADAS: ______ ! ______ " _____ #=!- "$

TOTAL Puntuacion MIR: _________ "-(#/3)

Nombre: ___________________________ Apellidos: ___________________________ Fecha: ____ / _______________ / 2002

10

11

12

13

14

15

16

17

18

19

20

3
21

1
22

4
23

2
24

3
25

5
26

1
27

3
28

2
29

2
30

4
31

1
32

3
33

2
34

2
35

5
36

5
37

5
38

3
39

1
40

4
41

4
42

4
43

5
44

1
45

4
46

4
47

4
48

2
49

3
50

4
51

2
52

4
53

1
54

4
55

4
56

4
57

4
58

4
59

1
60

4
61

2
62

5
63

4
64

2
65

5
66

1
67

3
68

2
69

4
70

1
71

4
72

2
73

4
74

2
75

4
76

1
77

1
78

2
79

4
80

2
81

2
82

1
83

5
84

5
85

2
86

1
87

2
88

4
89

3
90

5
91

1
92

2
93

1
94

5
95

2
96

3
97

2
98

2
99

5
100

2
101

5
102

2
103

5
104

3
105

3
106

4
107

3
108

5
109

4
110

5
111

3
112

1
113

4
114

3
115

2
116

1
117

1
118

4
119

5
120

5
121

3
122

4
123

2
124

1
125

1
126

3
127

2
128

1
129

4
130

4
131

2
132

3
133

5
134

5
135

5
136

2
137

2
138

5
139

3
140

3
141

4
142

2
143

3
144

5
145

4
146

5
147

4
148

5
149

1
150

3
151

4
152

4
153

2
154

4
155

1
156

4
157

2
158

5
159

4
160

5
161

2
162

1
163

3
164

2
165

4
166

4
167

4
168

1
169

4
170

2
171

3
172

2
173

5
174

4
175

3
176

2
177

4
178

5
179

1
180

5
181

3
182

2
183

1
184

3
185

1
186

4
187

1
188

3
189

2
190

3
191

3
192

2
193

1
194

1
195

5
196

1
197

1
198

2
199

4
200

3
201

5
202

5
203

5
204

5
205

4
206

4
207

5
208

3
209

5
210

3
211

2
212

2
213

3
214

3
215

1
216

2
217

2
218

3
219

5
220

5
221

4
222

1
223

2
224

3
225

4
226

5
227

2
228

1
229

1
230

1
231

5
232

2
233

4
234

4
235

5
236

5
237

2
238

5
239

1
240

3
241

5
242

3
243

4
244

4
245

5
246

3
247

2
248

4
249

2
250

1
251

5
252

4
253

3
254

5
255

1
256

1
257

1
258

5
259

5
260

Das könnte Ihnen auch gefallen